You are on page 1of 78

NCLEX QUESTIONS how to care for her mother appropriately.

Which nursing
diagnosis is most appropriate?
(1) Social isolation
1) The nurse makes the following entry on an elderly client’s
(2) Powerlessness
care plan, “Goal not met. Client refuses to ambulate, stating, ‘I
(3) Situational low self-esteem
am too afraid I will fall’”. The nurse should take which of the
(4) Altered role performance
following actions?
(1) Notify the physician
5) A 76-year-old client is admitted to the hospital. The nursing
(2) Reassign the client to another nurse.
assessment yields the following problems. Which of these
(3) Reexamine the nursing orders.
problems is related to aging?
(4) Write a new nursing diagnosis.
(1) Hyperopia
(2) Conductive hearing loss
2) Which of the following nursing diagnoses would most likely
(3) Presbycusis
be associated with the absence of hair on a 70-year-old male
(4) Tinnitus
client’s legs?
(1) Altered nutrition: less than body requirements.
6) Pain in the elderly requires careful assessment because older
(2) Risk for infection
people have which of the following characteristics?
(3) Fluid volume deficit
(1) Increased pain tolerance
(4) Tissue perfusion, altered: peripheral
(2) Decreased pain tolerance
(3) Are likely to experience chronic pain
3) A nurse is preparing to complete an admission assessment
(4) Experience reduced sensory perception
on a client that is partially hearing impaired. The best approach
would be to do which of the following?
7) A nurse is working in a geriatric screening clinic. The nurse
(1) Request that a family member be present.
would expect that the skin of the normal elderly client will
(2) Prepare written questions that cover the assessment criteria.
demonstrate which of the following characteristics?
(3) Speak slowly in a low-pitched voice while facing the client.
(1) Dehydration causing the skin to swell.
(4) Perform the physical assessment only at this time.
(2) Moist skin turgor
(3) Skin turgor showing a loss of elasticity.
4) An elderly, terminally ill client is being cared for by her
(4) Overhydration causing the skin to wrinkle
only daughter. The daughter expresses a fear of not knowing
8) A 78-year-old visually impaired client is admitted to the
nursing unit. Which of the following interventions would be Which of the following factors are most
likely to exacerbate risk for disease and
most appropriate in reducing sensory deprivation? frailty? (Select all that apply.)
(1) Close curtains to reduce glare. Loss of appetite and delirium
(2) Keep doors open to provide bright light into the room
Sleep disturbances and failures of self-care
(3) Keep curtains open to allow the sun to shine brightly in the
room Poor lifestyle choices and poverty
(4) Keep lights in the room dimmed. Iatrogenesis and dizziness

Risks for disease and frailty increase with age, and can be
9) Which of the following are associated with the aging exacerbated by poor lifestyle choices and poverty.
process. Choose all that apply.
(1) Increased visual impairment
(2) Increased calorie requirements
Which of the following groups of older
(3) Decreased sensation to heat and cold persons is more likely to have limitations
(4) Decreased subcutaneous fat in ADLs when chronically ill?
Caucasians are more likely than African-
Americans to have limitations in ADLs when
chronically ill.
Native Americans are more likely than
Caucasians to have limitations in ADLs when
Which of the following is an accurate chronically ill.
description of a frail older person? African-Americans are more likely than
A frail older person is one who is independent Caucasians to have limitations in ADLs when
in minimum activities of daily living. chronically ill.
A frail older person is one who has a Hispanics are more likely than African-
noticeable planned weight loss of 10 pounds in Americans and Caucasians to have limitations
a year. in ADLs when chronically ill.
A frail older person is one who exhibits
dependence in several activities of daily living. African-Americans are more likely than Caucasians to have
A frail older person is one who experiences limitations in ADLs when chronically ill.
frequent periods of depression.

A frail older person is one who exhibits dependence in one or more Gerontology nurses have the challenge to
activities of daily living. This dependency often restricts ADLs, and educate the older population and their
limitations in ADLs can contribute to feelings of depression. families that a healthy old age can be
achieved by: The gerontology nurse who is involved in
Taking advantage of recommended preventive
health promotion programs for older
health services and lifestyle changes.
persons living in the community and
Reducing their chances of disability with
vigorous exercises and balanced diet. diagnosed with Alzheimer's disease will
Becoming aware that cultural motivation is not focus on which of the following?
the key to achieving a healthy old age. Assisting in the ability to make appropriate,
A shift in paradigm as it relates to dietary informed choices
intake, rest, and yearly medical interventions. Achieving optimal mental and physiological
health
Americans can improve their chances for a healthy old age by Providing information and appropriate skills to
simply taking advantage of recommended preventive health maintain lifestyle changes
services and by making healthy lifestyle changes. Reducing genetic risk factors related to co-
morbidities of the older person

The established goal of the National When cognitive capacity is impaired due to the diagnosis of
Alzheimer's disease or another irreversible neurogenerative
Institute on Aging is to:
dementia, care and treatment decisions become even more
Provide appropriate healthcare to the
difficult. Treatment choices are not simple, because the burden of
chronically ill, so as to lengthen their years.
treatment and the success of needed interventions requires a
Decrease the significant burden of care placed
skilled interdisciplinary team approach.
on gerontology nurses and other healthcare
providers.
To "add life to years" by educating
professionals and supporting appropriate Which of the following classifications of
research. drugs would not directly cause delirium
To help promote sophisticated levels of through intoxication or withdrawal?
interdisciplinary care and "fine-tune" Anesthetics
appropriate therapies.
Analgesics
The established goal of the National Institute on Aging is to "add
Corticosteroids
life to years" and to educate professionals and support research
that establishes specific, practical ways to reduce disability. The Cardiotonics
National Institute on Aging does not provide direct healthcare
service to individuals. The agency's mission is to support and
Medications that can cause delirium through intoxication or
conduct high-quality research, to train and develop highly skilled
withdrawal include anesthetics, analgesics, anti-asthmatics,
research scientists, to develop and maintain state-of-the-art
anticonvulsants, antihypertensives, antimicrobials, corticosteroids,
resources, and to disseminate information to the public.
gastrointestinal H2 blockers, muscle relaxants, hypnotics, and
psychotropic medications.
Which of the following statements Healthcare providers can practice preventive ethics in order to
promote an environment where early identification of issues and
accurately describes modified aggressive
anticipation of possible dilemmas can serve to proactively avert
treatment for persons confined to nursing
potential areas of conflict. The knowledge of the values of the
homes? patient and family will allow the healthcare provider to anticipate
Modified aggressive treatment is usually where patient and family values might be in conflict.
appropriately delivered to older persons with
high functional ability.
Modified aggressive treatment is usually
appropriate for older persons with higher
degrees of frailty or multiple co-morbidities
Which of the following statements is accurate
who still have sufficient reserve capacity.
Modified aggressive treatment is appropriate
pertaining to the immune dysregulation theory?
for all older persons, and can be delivered
alone or in conjunction with aggressive or
modified care.
Modified aggressive treatment is delivered to
those with a life expectancy of six months or
less; patient's age is not a factor.

Modified aggressive treatment is usually appropriate for older he three characteristics that are unique to the immune system are
persons with higher degrees of frailty or multiple co-morbidities self-recognition, specificity, and memory. Antigens are a wide
who still have sufficient reserve capacity to respond to the range of substances that are identified as "non-self," and stimulate
demands of treatment. an immune response.

The three characteristics that are unique to the immune system


Because persons with end-stage dementia are self-recognition, specificity, and memory. Antigens are a wide
and physical frailty are in need of a range of substances that are identified as "non-self," and stimulate
Early identification of issues to help avert an immune response.
rational approach to care
potential areas of conflict. at the end of life,
preventive ethics to of
Having knowledge promote
the valuesa of
therapeutic
patient and
environment include: to healthcare.
family in relationship
Avoiding conflict resolution and The defense of natural immunity, or innate immunity, is not
communication skills with extended families. produced by the immune response that is present at birth.
Ongoing aggressive treatment of all disease Defenses of the innate immune system consist of a variety of
symptomatology to avert shortening of the physical barriers and biochemical and cellular defense mechanisms
elderly lifespan. to prevent the establishment of potential pathogens within the
body.
The cellular immune response is directly
Which of the following factors affect related to the T cell surface receptors.
proper immune system function? The humoral immune response is indirectly
Age, exercise related to the T cell surface receptors.
The cellular immune response is indirectly
Sunlight, nutrients in the diet related to the B cell surface receptors.
Diagnosis with co-morbidities The humoral immune response is initiated when an antigen binds
All of the above with antibody receptors on the surface of the mature B cell. In the
cell-mediated immune response, T cells bind to the foreign antigen
in multiple areas on the surface of the cell.
Aging results in lifetime accumulative effects of environmental
exposures, such as sunlight, radiation, pesticides, and the amount
of illness and stress the aged persons have been exposed to. A nurse is caring for a patient who is
deficient in T cells. The nurse would know
that T cell deficiency would result in which
The gerontology clinical nurse specialist is
of the following?
teaching a group of new graduate nurses
Hypersensitivity to allergens
how the body fights infection. When
discussing humoral immunity, the nurse Impaired cellular immunity
should explain that this protection is A decrease in the cell lifespan of granulocytes
mediated by which cell?
Basophil A cellular inability to participate in
phagocytosis
Eosinophil
T cells act directly with cellular targets, and are responsible for
The B cell cell-mediated immunity.
The monocyte

Which nursing intervention should be


The humoral immune response is initiated when an antigen binds
with antibody receptors on the surface of the mature B cell. attended to first for a client with a Type I
hypersensitivity reaction?
Administering naloxone (Narcan) stat
Which of the following statements are Obtaining blood sample for type and cross-
accurate about the humoral immune match
response and the cellular immune Initiating 0.9% NaCl intravenous solution
response? (Select all that apply.)
Establishing and maintaining a patent airway
The humoral immune response is directly
related to B cell surface receptors.
hypersensitivity that affects older people. Antibodies form against
Type I hypersensitivities are immediate, and can be life- the person's own IgGs, and the resulting complex is identified as
threatening. Manifestations include tightening of the throat, foreign.
shortness of breath, wheezing, tachycardia, and hypotension.

Which of the following classifications of


Which of the following clinical drugs are usually prescribed for patients
manifestations of a patient suggests Type with rheumatoid arthritis?
IV hypersensitivity reaction? Non-steroidal anti-inflammatory (NSAIDs)
Dermatitis from a latex allergy
Glucocorticoids
Sudden shortness of breath
Mineralocorticoid
The appearance of hives
Answers 1 and 2
Swelling of the lips

Drug therapy often consists of a combination of non-steroidal


Type IV hypersensitivity is also called delayed hypersensitivity. anti-inflammatory drugs (NSAIDs), anti-rheumatic drugs, and
Tissue is damaged as a result of a delayed T cell reaction to an glucocorticoids.
antigen. Examples of Type IV hypersensitivity include dermatitis
from a latex allergy, tuberculin reactions, and transplant
rejections. Which of the following are unique
characteristics associated with human
immunodeficiency virus (HIV) infection in
Under which of the following
the older person?
classifications of hypersensitivity would
HIV infection in the older person most likely is
systemic lupus erythematosus (SLE) and underdiagnosed and underreported.
rheumatoid arthritis be classified? Healthcare workers often do not take sexual
Type I hypersensitivity histories of older patients or recommend HIV
Type II hypersensitivity testing.
Symptoms of memory loss and weight loss
Type III hypersensitivity related to HIV often are treated as common
age-related health problems.
Type IV hypersensitivity Older persons are contacting HIV infection
through heterosexual and homosexual
Systemic lupus erythematosus is an example of type III activities.
hypersensitivity reaction caused by auto-antibody production. The
antibodies are formed against nuclear DNA and RNA throughout
the body. Rheumatoid arthritis is another example of type III
(B) delirium
The older person's susceptibility to (C) functional psychosis
infections is related to: (Select all that
(D) mania
apply.)
A decline in responsiveness of the immune (E) organic delusional syndrome
system to harmful foreign invaders.
The febrile response that signals infections 2. The percentage of community-dwelling individuals older
sometimes being blunted in the older person.
Medications commonly taken that also
than 65 years who require
decrease the normal fever response. assistance with bathing, dressing, walking within their homes,
The body temperature in older people being using the toilet, or eating is
approximately 1ºF lower than normal, closest to
meaning a rise in temperature can go
unnoticed.
(A) 10%
(B) 30%
UCLA Geriatrics Knowledge Test (C) 50%
DIRECTIONS: Each of the questions or incomplete statements (D) 70%
below is followed by suggested
answers or completions. Select the ONE that is BEST in each
case.
1. One day after operative repair of a left intertrochanteric 3. A 78-year-old nursing home resident has mild dementia
fracture of the femur, an 83-yearold man is agitated, confused, associated with Alzheimer’s
and appears to be having visual hallucinations. He has disease. She is disoriented to time and place but knows family
accused the nursing staff of trying to kill him. He is a widower members and regular nurse
and has lived alone for 11 aides by name. This patient’s capacity to make decisions
years. He has been forgetful for the past year but can perform regarding her health care is best
all activities of daily living determined by
without assistance. On mental status examination he is easily (A) mental status test
distracted, perseverates, and (B) her ability to understand treatment options
does not respond coherently to questions about orientation and (C) probate court decision
memory. The most likely (D) psychiatric examination
diagnosis is
(A) Alzheimer’s disease 4. Which of the following instruments is most appropriate to
assess physical functional
abilities in a 75-year-old nursing home resident? management is
(A) Activities of Daily Living Scale (A) observation
(B) Face/Hand Test (B) contact the family
(C) Instrumental Activities of Daily Living Scale (C) begin nasogastric tube feeding
(D) Zung Self-Rating Depression Scale (D) admit to the hospital
(E) perform percutaneous endoscopic gastrostomy
5. An 85-year-old man has a 4x7-cm Shea stage 3 pressure
ulcer over the sacrum. He has 7. A 78-year-old woman has right hemiplegia and dysarthria
been confined to bed since sustaining a hip fracture three after a cerebral infarction in the
months ago and has lost 10 lb left pons three weeks ago. She has been fed through a
during this time. Examination shows a foul-smelling wound, gastrostomy tube for two weeks
necrotic tissue covering 50% because of pneumonia of the left lower lobe, presumed to be
of the ulcer, and purulent drainage at the base. There is no secondary to aspiration. She
cellulitis. In addition to is depressed and would like to begin oral feedings.
surgical debridement, the most appropriate initial management Examination shows a decreased gag
is reflex and mild dysarthria. The most appropriate management
(A) topical application of neomycin/polymyxin/bacitracin is
ointment (A) continued tube feedings until gag reflex returns to normal
(B) topical application of sodium hypochlorite solution (B) encouragement to eat an appetizing diet
(C) application of an adhesive polyurethane dressing (C) ice chips orally and antidepressant therapy
(D) myocutaneous flap (D) supervised oral feeding of pureed foods
(E) none of the above (E) swallowing videofluoroscopy

6. An 82-year-old nursing home resident has had a 7-lb weight 8. The memory deficit that is most commonly caused by
loss over the past two months. depression is
She had a stroke seven years ago that resulted in aphasia. She (A) amnesia for traumatic events
is bed bound and needs (B) difficulty with free recall
assistance when eating. The staff reports that she often clamps (C) difficulty with multiple choice
her teeth and refuses to eat. (D) disorientation to person, place, and time
Physical examination shows no abnormalities. The most (E) symmetrical loss of multiple cognitive functions
appropriate next step in
9. Which of the following is best used for routine evaluation of Symptoms of obstruction, including hesitancy, straining, and
nutritional status in older intermittent urinary stream,
persons? have improved. He is otherwise healthy and takes no
(A) Delayed skin testing medications. Urinalysis shows
(B) Serial weight measurements normal findings. After voiding 220 mL without straining,
(C) Serum albumin level measurement postvoid residual volume is 40
(D) Serum transferrin level measurement mL. The most appropriate next step in management is
(E) Total lymphocyte count (A) oxybutynin 2.5 mg three times daily for a two-week trial
(B) terazosin 0.5 mg twice daily for a two-week trial
10. The most common community-acquired infection that (C) cystoscopy
results in hospitalization in older (D) cystometrography
persons is
(A) infected pressure ulcers 13. An 82-year-old woman with Alzheimer’s disease sustains a
(B) influenza displaced fracture of the
(C) pneumonia femoral neck in a fall. She resides in a nursing home because
(D) tuberculosis of her cognitive deficits.
(E) urosepsis Prior to the injury, she was independent in ambulation, feeding,
toileting, and transferring,
11. Approximately what percentage of the cost of long-term but needed cueing to bathe and dress. Her score on the Mini-
care is paid for directly by elderly Mental State Examination is
persons or their families? 11/30. The most appropriate management of the fracture is
(A) 30% (A) use of crutches and limited mobility
(B) 50% (B) traction and bed rest
(C) 70% (C) open reduction and internal fixation
(D) 90% (D) prosthetic replacement of the femoral head

12. An 82-year-old man has worsening of urge incontinence, 14. A 70-year-old man with diabetes mellitus and emphysema
urinary frequency, and nocturia comes to your office in
three months after transurethral resection of the prostate for December for influenza vaccination. You have cared for this
benign prostatic hyperplasia. patient for the past six
months and no additional medical records are available. He (E) Verification of need by two independent physicians
reports that prior to your care,
his last visit to a physician was 10 years ago. He also reports 17. What percentage of persons who are age 65 years and
that he received a tetanus older will require nursing home care
“shot” in the emergency department four years ago because of at some time during their life?
a laceration. Which of the (A) 5%
following is the most appropriate schedule for immunization? (B) 25%
(A) Influenza vaccine only at this visit (C) 45%
(B) Influenza and pneumococcal vaccines at this visit (D) 65%
(C) Influenza vaccine at this visit and pneumococcal vaccine (E) 85%
in eight weeks
(D) Pneumococcal vaccine at this visit and influenza vaccine 18. A 79-year-old woman has had confusion since admission to
in eight weeks the hospital for pneumonia one
week ago. Her prior cognitive status is unknown; she appeared
15. A 76-year-old woman has pain in the right hip and is malnourished and unkempt
unable to walk without support three on admission. She has responded well to antibiotic therapy and
days after a fall. Passive motion of the right hip produces pain is currently afebrile. She
in the groin. Plain is pleasant and attentive to questions. Mental status
radiographs of the pelvis show no abnormalities. The most examination reveals disorientation,
likely cause of her symptoms is memory impairment, and language abnormalities; she is unable
(A) muscle strain to draw the face of a clock.
(B) occult fracture of the hip The remainder of the physical and neurologic examination
(C) osteoarthrosis shows no abnormalities. The
(D) referred pain from the spine most likely diagnosis is
(A) age-associated memory loss
16. Which of the following is required for eligibility for the (B) delirium
Medicare Hospice Benefit? (C) dementia
(A) Diagnosis of a terminal malignancy (D) depression
(B) Enrollment in Medicare Part A (E) paraphrenia
(C) Forfeiture of all Veterans’ benefits
(D) Supplemental insurance policy
19. A 68-year-old man has a moderate left hemiparesis and (E) noncompliance
severe neglect six days after a right
hemispheric stroke. He appears to be in denial about his 21. An otherwise healthy 73-year-old man with Parkinson’s
degree of impairment and states disease has difficulty walking. He
that he hopes to be discharged from the acute hospital to his has had many changes in his antiparkinsonian medication and
home where he has lived alone feels that the present
for the past 10 years. During physical therapy sessions, he is regimen is the best he has ever had. Analysis of gait reveals
apathetic and tires easily after very slow step initiation,
30 minutes. Further evaluation shows no abnormalities. decreased stride length with shuffling, and moderate
Which of the following is the retropulsion. Which of the following
most appropriate site for rehabilitation? devices is most likely to improve his gait?
(A) Home-care program (A) A straight cane
(B) Outpatient physical therapy program (B) A four-prong quad cane
(C) Acute inpatient rehabilitation program (C) A walker with arm troughs attached
(D) Nursing home (D) A pick-up four-legged walker
(E) Inpatient geropsychiatric unit (E) A front-wheeled walker
Items 22-23
20. An 84-year-old man is fitted for and trained in the use of a For each patient, select the most appropriate management (A-
hearing aid. Three months later, L).
he reports that he has stopped using the hearing aid because, (A) Pelvic muscle exercises
although he can hear better, he (B) Prompted voiding
still cannot understand what people are saying. He states that (C) Biofeedback
background noise is also a (D) Urodynamic assessment
problem. He has hypertension but is otherwise healthy. His (E) Oxybutynin 2.5 mg three times daily
score on the Mini-Mental (F) Phenylpropanolamine 75 mg twice daily
State Examination is 30/30. The most likely cause of the (G) Vaginal estrogen daily
hearing difficulty is (H) Terazosin 0.5 mg twice daily
(A) central auditory processing disorder (I) Bethanechol 10 mg three times daily
(B) early signs of dementia (J) Bladder neck suspension
(C) improper fit of the hearing aid (K) Intermittent catheterization
(D) Meniere’s disease (L) Chronic indwelling catheterization
22. An 82-year-old woman is admitted to a nursing home two 3. The progression of intermittent claudication is measured by
weeks after operative repair of a the:
hip fracture. She has an indwelling catheter for recurring 1. distance walked before leg pain starts.
postoperative urinary retention. 2. pedal edema after dangling the legs for 20 minutes.
3. peripheral pulses in the affected leg.
23. An 89-year-old woman with Alzheimer’s disease, mild 4. skin temperature and color of the feet.
congestive heart failure, and Correct Response: 1
glaucoma has urinary incontinence several times daily,
occurring both day and night and 4. An early sign of alcohol withdrawal is:
without residual urine. She is wheelchair-bound and resides in 1. auditory hallucinations.
a nursing home. 2. decreased blood pressure.
3. depression.
4. diaphoresis.
Correct Response: 4
1. For an individual with age-related hearing loss, which sound 5. A stage III pressure ulcer is characterized by:
is most difficult to hear? 1. blisters, abrasions, or shallow craters.
1. A recording of a march played softly 2. deep craters with or without undermining and full-thickness
2. A young child talking in a cafeteria line skin loss involving
3. Hammering during construction of a house next door subcutaneous tissue.
4. The voice of a man speaking in an elevator 3. full-thickness skin loss with tissue necrosis or damage to
Correct Response: 2 muscle or bone.
4. partial-thickness skin loss involving the dermis or epidermis.
2. Pain of gastrointestinal origin is best differentiated from pain Correct Response: 2
of cardiac origin by the
presence of: 6. The primary risk factor for the development of pressure
1. chest pain lasting longer than five minutes. ulcers in older adult patients is:
2. chest pain of rapid onset. 1. immobility.
3. left flank pain. 2. impaired circulation.
4. substernal chest discomfort. 3. incontinence.
Correct Response: 4 4. malnutrition.
Correct Response: 1 4. oxygen therapy at 3 L/min as needed and deep-breathing
exercises for
7. An 80-year-old resident of a retirement center states that relaxation.
something is wrong with the Correct Response: 1
lighting in the room because colored rings appear around the
light bulbs. The resident 10. A 65-year-old man with a 45-year history of smoking
most likely has: complains of a change in his
1. cataracts. cough pattern, a nonproductive cough, and an ache in his chest.
2. delusions. The man's chest x-ray
3. glaucoma. reveals an infiltrate. The gerontological nurse suspects:
4. increased intracranial pressure. 1. a lung tumor.
Correct Response: 3 2. chronic obstructive pulmonary disease.
3. pulmonary edema.
8. The most common cause of chronic pain in older adults is: 4. tuberculosis.
1. arthritis. Correct Response: 1
2. fractures. 11. Which symptom in older adults is most indicative of a
3. headaches. urinary tract infection?
4. neuropathy. 1. Confusion
Correct Response: 1 2. Dysuria
3. Fever
9. A 72-year-old man with asthma, chronic obstructive 4. Frequency
pulmonary disease, and chronic Correct Response: 1
anxiety is admitted to a nursing home. Care plan objectives for
this man include: 12. Which is a risk factor for vaginitis in older adult women?
1. adherence to his medication regimen, inhalation therapy, and 1. Anticoagulation therapy
instruction about 2. Increased sexual activity
methods of conserving energy. 3. Poor nutrition
2. an exercise program to increase the vital capacity of his 4. Prolonged antibiotic therapy
lungs. Correct Response: 4
3. instruction in respiratory exercises with emphasis on forced
inhalation.
13. Which condition might be indicated by an increase in Correct Response: 1
hemoglobin values? 1. Dehydration
2. Infection 17. Heat stroke is a serious form of hyperthermia that is
3. Malnutrition characterized by:
4. Opiate use 1. absence of sweating.
Correct Response: 1 2. decrease in body temperature.
3. increase in sweating.
14. An 83-year-old female patient underwent a total hip 4. nausea and vomiting.
replacement three days ago. She Correct Response: 1
should not cross her legs because:
1. abduction of the hip can cause dislocation of the prosthesis. 18. A gerontological nurse is teaching dressing techniques to a
2. adduction of the hip can cause dislocation of the prosthesis. patient with right hemiplegia
3. blood clots often result from pressure on arteries in the legs. and a severe speech impairment. An appropriate first step in
4. contracture can be prevented by avoiding acute flexion of the nurse's teaching would
the hip. be to:
Correct Response: 2 1. ask the patient to put on a shirt.
2. demonstrate the proper way to put on a shirt.
15. A 76-year-old patient with osteoarthritis complains of pain, 3. explain the difficulties in putting on a shirt.
stiffness, and deformities of 4. give verbal instructions on dressing procedures.
the fingers. The gerontological nurse recommends: Correct Response: 2
1. cold packs.
2. exercise. 19. Older adults with Parkinson disease exhibit:
3. meditation therapy. 1. confusion and depression.
4. vitamin therapy. 2. dementia and hyperkinesia.
Correct Response: 2 3. rigidity and tremor at rest.
4. weakness and tremor with movement.
16. A factor that contributes to hypothermia in older adults is: Correct Response: 3
1. decreased activity.
2. decreased vulnerability to cold. 20. To prevent injury, a gerontological nurse advises an older
3. increased perception of cold. adult who is taking tricyclic
4. increased subcutaneous fat. antidepressants to:
1. eat a diet high in roughage. 24. A 78-year-old patient calls a telephone triage nurse and
2. get an additional night light. reports several falls after taking
3. provide lubrication for the oral mucosa. over-the-counter medication for a recent cold. Which
4. stand up slowly from sitting or lying positions. medication contributed to the falls?
Correct Response: 4 1. Diphenhydramine (Benadryl)
2. Ferrous sulfate
21. Persons who are taking anticholinergic drugs are at high 3. Guaifenesin (Robitussin)
risk for: 4. Loratadine (Claritin)
1. cardiac arrhythmias. Correct Response: 1
2. dry mouth.
3. orthostatic hypotension. 25. Older adults who take the herbal supplement cascara
4. slurred speech. sagrada are monitored for the
Correct Response: 2 presence of:
1. fever.
2. hypokalemia.
22. Age-related changes in which two organs most affect an 3. jaundice.
older adult's reaction to 4. vertigo.
medication? Correct Response: 2
1. Heart and lungs
2. Intestines and spleen 26. A nursing home conducts a survey to evaluate nursing care.
3. Liver and kidneys However, some residents
4. Pancreas and gall bladder did not express their concerns due to fear of reprisal by the
Correct Response: 3 staff. Which aspect of the
evaluation is most affected?
23. Sildenafil citrate (Viagra) is hazardous for patients with: 1. Generalizability
1. a history of coronary artery bypass graft. 2. Reliability
2. a history of shingles. 3. Statistical significance
3. heart failure and borderline hypotension. 4. Validity
4. Paget disease and hypertension. Correct Response: 4
Correct Response: 3
27. A common side effect of angiotensin-converting enzyme
inhibitors that frequently results 30. Reminiscence therapy promotes an older adult's sense of
in discontinuation of therapy is: 1. a dry, persistent cough. security by:
2. exacerbation of heart failure. 1. increasing socialization skills.
3. sedation. 2. meshing the past with the future.
4. urinary incontinence. 3. providing praise and recognition.
Correct Response: 1 4. reviewing comforting memories.
Correct Response: 4
28. A patient with Stage 2 Alzheimer disease visits the mental
health clinic. During the 31. A physician has just informed an older adult patient that
interview, the patient becomes hostile and refuses to answer test results indicate that the
further questions. The patient has cancer and will require extensive surgery. The
gerontological nurse's best action is to: patient says, “I know the
1. ask if the questions upset the patient in any way. tests are wrong. I feel fine.” The gerontological nurse's most
2. discontinue the interview. appropriate response is to:
3. explain that the information is needed to plan the patient's 1. acknowledge that the patient looks healthy and encourage
care. seeking a second
4. ignore the patient's reaction and proceed. opinion.
Correct Response: 2 2. advise the patient to join a support group.
3. convey availability to talk to the patient. 4. tell the patient
29. A gerontological nurse at a nursing home conducts a that the tests are reliable and accurate.
reminiscence therapy group for Correct Response: 3
residents with confusion. A member of the group stands up and
says, “I just heard my 32. A 75-year-old patient who sustained a stroke has residual
cow. I have to go and milk her now.” The nurse's most left-sided weakness. From
therapeutic response is: the first day of hospitalization, the patient has been combative
1. “All right, you may leave the group now.” and demanding, and has
2. “Please tell us about your cow.” refused to swallow any medication. The most constructive
3. “That wasn't a cow; maybe you heard a vacuum cleaner.” nursing action is to:
4. “You live here at the nursing home now, not on the farm.” 1. continue to attempt to follow the physician's orders.
Correct Response: 2 2. determine the patient's premorbid personality.
3. restrain the patient and request a change in the route of The patient does not use alcohol, caffeine, or any medications
medication. other than
4. wait for the patient to become more cooperative. acetaminophen for arthritis pain. The patient goes to bed at
Correct Response: 2 11:00 pm, falls asleep in 15
minutes, awakens several times during the night, and promptly
33. A healthy 80-year-old female patient complains that her goes back to sleep. The
skin feels dry and sometimes patient feels refreshed in the morning and works five days a
itchy. The gerontological nurse advises her to: week as a volunteer. The
1. avoid scratching since breaks in the skin increase the risk of gerontological nurse's most appropriate response is to:
infection. 1. recommend that the patient eliminate fluid intake after 6:00
2. drink more liquids and take showers instead of baths. pm.
3. take fewer baths, use soap sparingly, and apply skin cream 2. recommend that the patient go to bed one hour earlier. 3.
afterward. suggest that the patient enroll in a sleep study.
4. wear cotton clothing and try a different brand of soap. 4. tell the patient that the sleeping pattern is a normal age-
Correct Response: 3 related change.
34. A 90-year-old patient with multiple medical problems is Correct Response: 4
admitted to the hospital's 36. Which signs are characteristic of deep vein thrombosis?
geriatric care unit. The nursing assessment reveals lethargy, 1. A cool, non-tender limb
poor capillary perfusion, 2. Limb numbness with diaphoresis
and urinary incontinence. These findings alert the 3. Rapid onset of unilateral leg swelling with dependent edema
gerontological nurse to the potential 4. Vertigo with an abrupt onset of blurred vision
for: Correct Response: 3
1. aspiration.
2. contractures. 37. Unusual symptoms of pneumonia often found in long-term
3. dehydration. care residents include:
4. skin breakdown. 1. anorexia and new behavioral problems.
Correct Response: 4 2. headache and difficulty breathing.
3. muscle aches and fever.
35. An 80-year-old patient complains of sleeping less despite 4. nonproductive cough and chest pain.
spending more time in bed. Correct Response: 1
38. Which symptom is exhibited first by an older adult with a 4. restore the patient's fluid and electrolyte balance.
urinary tract infection? Correct Response: 4
1. Anorexia
2. Confusion 42. The adult children of an aging couple ask a gerontological
3. Fever nurse about alternative
4. Restlessness therapies or nontraditional services to improve their parents'
Correct Response: 2 health. The nurse's most
appropriate response is to:
39. Which assessment tool measures economic resources, 1. discuss the benefits, risks, and limitations of various
mental health, and activities of therapies.
daily living? 2. distinguish between folk and traditional medicine.
1. Lawton's Physical Self-Maintenance Scale 3. give a firm warning about alternative therapies.
2. Older Americans Resources and Services Assessment 4. recommend a reputable holistic health therapist.
3. Problem Oriented Medical Record Correct Response: 1
4. Short Portable Mental Status Questionnaire
Correct Response: 2 43. A 73-year-old patient is admitted to a rehabilitation facility
40. All care plans for older adult patients include: after sustaining a mild stroke.
1. a bowel and bladder program. After three nights in the facility, the patient begins to sleep
2. a fall prevention program. 3. discharge planning. only four to five hours a night
4. reminiscence therapy. and to awaken frequently during the night. The patient then
Correct Response: 3 complains of not feeling
rested and begins to nap during the day. Which is the most
41. An older adult patient, who is recovering from surgery, has appropriate nursing action?
a sodium level of 128 mEq/L 1. Completing an assessment of the patient's sleep-wake cycle
and is confused. The physician diagnoses syndrome of to determine
inappropriate secretion of necessary interventions
antidiuretic hormone. The gerontological nurse's primary goal 2. Doing nothing since this type of sleep pattern is associated
for this patient is to: with normal aging
1. decrease edema by restricting free water intake. 3. Inquiring if the patient takes a medication at bedtime and
2. prevent complications of hyponatremia. requesting that the
3. reorient the patient to his or her surroundings. physician order it
4. Moving the patient further away from the nurse's station to 2. move him as quickly as possible so that he does not have
minimize disturbances time to think.
Correct Response: 1 3. restrict family visits for the first two weeks to give him time
to adjust.
44. A 68-year-old man calls his daughter every night to talk 4. suggest that he bring his favorite things from home to make
about his beloved wife who died his room seem
four weeks ago. During the day, he is sad and goes out familiar.
frequently to get away from the Correct Response: 4
empty house. The man's most probable state is:
1. depression. 2. mourning. 47. An 80-year-old patient is in the terminal stage of Alzheimer
3. neurosis. disease. The treatment team
4. prolonged grief. meets with the patient's who are angry and complain about a
Correct Response: 2 recent incident in which
their parent's dentures were misplaced. The team members
45. The main reason that older adults with a chronic illness realize that:
tolerate functional impairments 1. anger at staff is a symptom of grief and needs to be
is that they: addressed.
1. are afraid to seek medical advice for fear of what they may 2. anxiety about the meeting may have interfered with the
find. children's affect.
2. associate their symptoms with aging rather than an illness. 3. material items are the focus for the children at this time.
3. believe in home remedies and parental traditions. 4. the focus of control should shift to the parent.
4. relate their symptoms to conditions that resolved in the past Correct Response: 1
without treatment.
Correct Response: 2 48. When relocating from a family home to a continuing care
community, the most crucial
46. An 87-year-old man, who has been living independently, is factor in an older person's adjustment is his or her:
entering a nursing home. To 1. level of economic independence.
help him adjust, the most effective action is to: 2. perceived control of the move.
1. involve him in as many activities as possible so he can meet 3. physical proximity to remaining family.
other residents. 4. risk-taking ability.
Correct Response: 2
2. changing the topic when a patient talks about his or her
49. A 78-year-old male resident at a long-term care facility, morbid past.
who is a former business 3. encouraging reminiscence, oral histories, and storytelling.
executive, has been smoking and extinguishing cigarettes in a 4. helping the older adult explore how spiritual involvement
paper cup in areas assists with stress relief.
where smoking is prohibited. He has been informed repeatedly Correct Response: 3
of the designated
smoking areas. The resident's behavior indicates an attempt to: 52. The holiday season is approaching and a woman is
1. express self-transcendence. admitted to a psychiatric unit. She
2. maintain autonomy by exercising control. reports that her husband of 45 years passed away four weeks
3. maintain his previous professional role. ago. She frequently cries,
4. react against the facility's ageism. eats poorly, periodically complains of back and stomach aches,
Correct Response: 2 and has begun isolating
herself. What type of grief is this woman exhibiting?
50. A gerontological nurse is caring for an older adult who has 1. Acute
been confined to home for 2. Anticipatory
the last 10 weeks due to illness. The patient is anxious, has 3. Disenfranchising
multiple somatic 4. Dysfunctional
complaints, and has become unable to follow instructions. The Correct Response: 1
nurse knows that this
phenomenon commonly occurs with: 53. A 92-year old patient, who recently underwent a below-the-
1. environmental overload. knee leg amputation, is
2. protective isolation. resisting attempts at rehabilitation. The most likely reason for
3. selective inattention. the resistance is that the
4. sensory deprivation. patient:
Correct Response: 4 1. has goals that differ from the rehabilitation care plan.
2. has too many disabilities to realistically plan for
51. The gerontological nurse facilitates the benefits of life rehabilitation.
review by: 3. is too old to undergo rehabilitation.
1. assisting the older adult to accept death as the inevitable last 4. requires a psychiatrist's assistance to deal with the loss.
stage. Correct Response: 1
1. Becoming defensive when questions are asked
54. When the gerontological nurse in a clinic asks a 70-year- 2. Complaining about care delivered by hospital staff
old man about his sexual 3. Giving an illogical account of her mother's fall
activity, he begins to cry and says, “I feel so bad for my wife; 4. Refusing to leave her mother alone to answer questions
she is only 60, and I can't Correct Response: 4
seem to satisfy her as often as I used to.” Which is the nurse's
most appropriate 56. In teaching an older adult client, the gerontological nurse's
response? most appropriate initial
1. “At your age, sexual activity diminishes because of changes strategy is to:
in your circulation. I 1. assess the client and individualize the teaching methods.
will explain this to your wife.” 2. set a slow learning pace and begin teaching simple concepts.
2. “Certain body functions, such as erections, slow down with 3. teach slowly and use repetition.
age. Could you tell 4. use demonstration and provide ample opportunity for
me more about your sexual relations? For instance, how often practice.
do you have Correct Response: 1
intercourse?”
3. “Your problem is probably an emotional one. If you could 57. In preparing a presentation for older adults, a
relax, you would be as gerontological nurse keeps in mind that:
sexually active as you were 10 years ago.” 1. older adults are unlikely to participate in educational
4. “Your problem is probably due to a decrease in your sexual programs due to lack of
hormones. This interest.
occurs naturally as people age.” 2. older adult learners are heterogeneous due to diverse
Correct Response: 2 educational experiences
and learning strategies.
55. An alert and oriented 82-year-old woman, who lives with 3. the ability to acquire knowledge from a verbal presentation
her daughter, has been decreases with age
admitted to the hospital with bruises about the face and head. more than the ability to acquire knowledge through reasoning.
The daughter reports that 4. the age-related decline in intellectual performance creates
her mother fell. Which behavior by the daughter raises the obstacles for acquiring
greatest suspicion of elder new information.
abuse? Correct Response: 2
4. is stimulating so as to challenge thought.
58. Members of a family are caring for their father at home. Correct Response: 2
Which statement by a family
member indicates a need for teaching and caregiver 61. In assessing the lighting for a patient with glaucoma, the
instruction? gerontological nurse knows
1. “Dad has gotten lazy about his bathroom habits. He blames that:
his arthritis 1. ceiling lights are best.
medication for his toileting accidents.” 2. drapes should be left open during the daylight hours.
2. “Dad's room is close to the bathroom and we keep a light on 3. higher levels of light are needed.
for him at night.” 4. lower levels of light are needed.
3. “It's inconvenient, but we stop other activities to remind Dad Correct Response: 4
to go to the 62. A 63-year-old patient is returning home after being
bathroom on a regular schedule.” hospitalized for injuries received
4. “We try to avoid coffee and tea at night, but Dad really likes during a robbery and home invasion. Although neighborhood
a cup of coffee for robberies are rampant, the
breakfast.” patient has lived in the same house for 50 years and does not
Correct Response: 1 want to move. The
patient receives a monthly social security check. The
59. When hospital quality assurance indicators are assigned, gerontological nurse's most
the nursing staff strives to: appropriate step is to:
1. avoid emphasizing performance deficits. 1. advise the patient to arrange for someone to visit regularly or
2. decrease patient falls. move into the
3. focus on achieving 100% compliance. home.
4. identify high-risk, high-volume, or problem-prone areas. 2. advise the patient to have the social security check deposited
Correct Response: 4 directly to the bank
and to get a dog.
60. The most appropriate environment for a person with 3. assume that the patient is incompetent and initiate
chronic dementia is one that: commitment proceedings.
1. changes often to decrease boredom. 4. take no action since the patient has a right to autonomy.
2. contains familiar objects. Correct Response: 2
3. is limited in color and sound.
63. A gerontological nurse in a daycare program for older 2. Hip
adults observes that the 3. Shoulder
participants have long toenails, corns, calluses, and other 4. Vertebral
problems indicating a need Correct Response: 2
for better foot care. What is the nurse's best action?
1. Developing an educational program on foot health and 66. The primary reason for establishing quality improvement
arranging for podiatry committees at long-term care
services at the site facilities is to:
2. Establishing a regular foot care plan whereby the 1. facilitate staff participation.
participants' toenails would be 2. initiate changes based on interdisciplinary exchange.
cut and corns and calluses shaved 3. monitor and record incidents, accidents, and injuries.
3. Instructing competent family members in the proper 4. provide quality care based on measurable data.
methods of cutting toenails Correct Response: 4
and using commercial foot care products.
4. Recommending that the participants soak their feet for 10 67. An accrediting body evaluates a nursing home by
minutes before cutting monitoring the number of residents
their toenails using safe toenail clippers who developed pressure ulcers and urinary tract infections.
Correct Response: 1 What type of audit is being conducted?
1. Outcomes
64. The occurrence of tuberculosis in the older adult is 2. Process
significantly increased among 3. Prospective
individuals who 4. Structure
1. are physically inactive. Correct Response: 1
2. are cigarette smokers.
3. have received the Bacile Calmette Guérin (BCG) vaccine. 68. A gerontological nurse is creating a staff development
4. reside in institutions. program for a unit. To assess the
Correct Response: 4 staff's learning needs, the nurse's best approach is to ask staff
members:
65. Which type of fracture has the highest morbidity and 1. “How do you want to learn new material?”
mortality in the older adult? 2. “What do you need to know to do your job better?”
1. Ankle 3. “What do you think others need to learn?”
4. “What do you think others want you to learn?” 72. The gerontological nurses in a teaching nursing home are
Correct Response: 2 informed of a research
project to obtain voiding cystometrograms on all residents. The
69. A comprehensive staff development program in a long- gerontological nurses'
term care facility is based on the: responsibility is to ensure that the:
1. availability of educational resources. 1. non-English speaking residents receive a complete
2. nursing director's perceptions of staff learning needs. explanation of the study in
3. philosophy, goals, and objectives of the organization. their native languages.
4. recommendations of the ombudsman. 2. nursing home administration know that the nurses do not
Correct Response: 3 support such invasive
70. A gerontological charge nurse delegates the administration studies on the residents.
of a nasogastric tube 3. research team is well staffed so that the nursing home staff
feeding to a licensed practical nurse (LPN). Which statement can provide care as usual.
about this situation is most 4. residents will be properly medicated in the examining room
accurate? where the tests will be conducted.
1. The charge nurse is responsible for delegated care. Correct Response: 1
2. The charge nurse should implement the care and not
delegate the task. 73. Which is an accurate statement about Medicare?
3. The LPN is accountable for his or her own actions. 1. Medicare is a health insurance program with carefully
4. The LPN should respectfully refuse to initiate this care. described benefits that may restrict the length of hospital stays.
Correct Response: 1 2. Medicare patients can expect to receive routine household
and attendant care under Part A of their Medicare insurance.
71. Which profession first developed standards of 3. Medicare pays for the majority of nursing home care for
gerontological care and provided a persons who are 65 years of age or older.
certification mechanism to ensure expertise? 4. Persons covered by Medicare must pay once-in-a-lifetime
1. Medicine deductibles on both the physician and hospital portions of the
2. Nursing program.
3. Physical therapy Correct Response: 1
4. Social work
Correct Response: 2 74. Nursing facilities that receive federal funds must complete
for all residents a:
1. minimum data set and care plan within seven days. The gerontology nurse practitioner should
2. resident assessment instrument and care plan on admission. rule out (R/O) which symptom before the
3. resident assessment instrument and care plan within 14 days. diagnosis of dementia is made?
4. resident assessment instrument within 14 days and a care History of seizures (convulsions)
plan within 21 days. History of head injury
Correct Response: 4 History of schizophrenia

History of delirium
75. The primary function of an ombudsman is to:
1. act as a clearinghouse for complaints and problems.
Because cognition impairment caused by delirium might be
2. initiate complaints about the facility's operations. reversible, delirium must be ruled out when diagnosing dementia.
3. question the facility administrator and the director of
nursing.
4. welcome and interview patients and their families. The definitive medical diagnosis of AD is
Correct Response: 1 confirmed by:
Histopathological findings.

History and physical examination.

Neuropsychological testing.

Combing estimates of genetic and


environmental risk factors.

The definitive diagnosis of AD is provided only after


histopathological confirmation at autopsy. However, clinical
diagnosis can be made from history, physical examination, and
neuropsychological testing.

Which of the following statements is


The majority of cases of Alzheimer's disease (AD) result from accurate pertaining to frontotemporal
complex interactions between genetic and environmental factors. dementia? Frontotemporal dementia:
Studies have shown that the major risk factor for developing AD is (Select all that apply.)
the Σ4 allele of the apolipoprotein (APOE) on chromosome 19. Includes Lewy bodies, and is diagnosed on the
basis of personality changes.
Is an abrupt-onset dementia with abnormal
reflexes and emotional ability.
Includes Pick's disease, and is diagnosed on Carbenicillin Indanyl sodium (Geocillin)
the basis of personality changes and the
presence frontal brain atrophy. Carboplatin (Paraplatin)
Includes Lewy bodies, Pick's inside nerve Carisoprodol (Rela)
cells, and behavioral disinhibition.

Frontotemporal dementia, which includes Pick's disease, is The pathology of PD is related to the loss of dopaminergic cells
diagnosed on the basis of personality changes and the presence of situated deep in the midbrain in the substantia nigra. Dopamine is
frontal brain atrophy. necessary for neurotransmission of nerve terminals, which is
involved in the initiation of movement. Levodopa is the metabolic
precursor of dopamine that provides symptomatic relief of some of
PD symptoms, especially bradykinesia. Levodopa is combined with
Carbidopa, which is a peripheral dopa-carboxylase inhibitor.

Nurses who administer Sinemet as


prescribed for the elderly should
Parkinsonian disease (PD) is referred to as incorporate timing of the administration of
an extrapyramidal syndrome because it Sinemet into the patient's care plan and:
manifests which of the following clinical Give Sinemet 30 minutes after meals.
symptoms?
Tremor and bradykinesia Give Sinemet 30 minutes before meals.

Diarrhea and sweating Give Sinemet one hour before meals.

Constipation and hypotension Give Sinemet with meals, followed by a glass


of water.
Somnolence and poor gait
Sinemet should be taken on an empty stomach. The nurse should
incorporate timing of the administration of Sinemet into the
Parkinsonian disease (PD) has variable combinations of signs and
patient's plan of care. Both the patient and caregiver should be
symptoms that include tremor, rigidity, bradykinesia, and
taught that Sinemet should be taken one hour before or two hours
disturbances in gait.
after a meal.

Which of the following drugs would be


The pathology of a stroke typically is not
ordered for persons with Parkinsonian
caused by which of the following?
disease? Hemorrhage into the brain
Carbidopa/Levodopa (Sinemet)
An embolus or thrombus that occludes an
artery Measuring the patient's height and weight
Rupture of an extracerebral artery, causing accurately.
subarachnoid hemorrhage
Damage to the cerebrum and myelin sheath There are several nursing interventions when caring for a patient
with a history of seizures. It is the nurse's responsibility to obtain
an accurate patient history, including age of seizure onset and
The pathology of a stroke typically is caused by hemorrhage into frequency of attacks. It is important for the nurse to be aware of
the brain, an embolus or thrombus that occludes an artery, or the medication dose, because many patients are given a starting
rupture of an extracerebral artery, causing subarachnoid dose, which might need to be titrated.
hemorrhage.

The most important responsibility of the


Which prescription most likely would be
nurse when a patient has a seizure is to:
ordered if not contraindicated for acute Maintain the head-tilt-chin lift method.
ischemic stroke?
Packed red blood cells (PRBC), followed by Provide oxygen immediately.
furosemide (Lasix)
Prevent injury of the patient.
25% albumin followed by 0.9% sodium
chloride Prevent status epilepticus.
Recombinant-tissue plasminogen activator (rt-
PA)
If a patient has a seizure, the most important responsibility of the
Heparin infusion alternating with coumadin
nurse is to prevent injury to the patient. Oxygen should always be
(Warfarin)
available, and the head-tilt-chin method can be used to obtain an
If thrombolytic therapy is not contraindicated, then recombinant airway.
tissue plasminogen activator (rt-PA) often will be administered
within three hours to treat the acute ischemic stroke. However, not
all patients treated with rt-PA therapy will have positive outcomes.

Nursing interventions for persons with


history of seizure would include: (Select
all that apply.)
Patient-family education in regards to
Obtaining an accurate patient history.
seizures would include: (Select all
Knowledge of the frequency of attacks. that apply.)
Providing patient and family with
Being aware of the medication dosage and
audiovisual aids for ongoing review.
frequency.
Teaching how to identify and avoid
precipitating factors for seizures.
Stressing the need to take medication as
prescribed.
Teaching what to do in the event a
seizure occurs.
Small, frequent feedings requires less release of hydrochloric acid.
Remaining in an upright position for 30 minutes after meals
Which of the prevents reflux into the esophagus.
following are
aged-related
changes that Decline in
sense of Persons with GERD should be referred to a primary care provider
affect for a thorough cardiac evaluation to rule out cardiac disease. The
taste and
gastrointestinal smell, most frequently used diagnostic test is barium swallow. Upper
function? decrease in endoscopy is the best method to assess mucosal injury. Acid
(Select all that salivary perfusion tests usually are not necessary, and require the
apply.) secretion placement of an esophageal probe above the esophageal sphincter
Diminished to collect esophageal contents.
capacity of
the gastric
mucosa to
resist such Bed-bound patients should not be fed in the semi-reclined position
factors as or fed thin liquids that might quickly drain into the esophagus
Helicobacter before the swallow reflex is triggered. A heavy caseload of clients
pylori might cause healthcare providers to rush-feed patients, resulting
Achlorhydria in reflux of nutrients and gastric acid.
or insufficient
hydrochloric
acid in the
Stroke, especially in the midbrain and anterior cortical areas, is the
stomach
most common cause of dysphagia in the older person. Parkinson's
Decreases in
disease and Alzheimer's disease can cause dysphagia by inhibiting
intestinal
movements of the tongue, pharynx, or upper esophagus. Crohn's
absorption,
and ulcerative colitis affect the G.I. mucosa and submucosa, and
motility, and
do not usually cause esophageal reflux or aspiration.
blood flow

Preparation of the videofluoroscope radiographic evaluation


includes drinking of a radio opaque solution, which is a chalk
mixture. The chalk mixture is constipating, and could cause fecal Older patients with GERD and Barrett's esophagus require
impaction. aggressive treatment with proton pump inhibitors and regular
endoscopic examination. If the esophageal erosion does not
reverse with treatment, the surgery of choice is Nissen's
Which of the following statements are fundoplication, and involves closing any hiatal hernia and restoring
accurate as they relate to medications an antireflux barrier.
used to manage GERD? (Select all that
apply.)
Magnesium-containing antacids can cause Which of the following types of gastritis is
diarrhea. associated with Helicobacter pylori and
Aluminum-containing antacids can cause duodenal ulcers?
constipation. Erosive (hemorrhagic) gastritis
Cimetidine (Tagamet HB) causes osteomalacia
Fundic gland gastritis (type A)
and hypophosphatemia.
Misoprostol's (Cytotec) major side effect is Antral gland gastritis (type B)
G.I. bleeding.
Aspiring-induced gastric ulcer
Magnesium-containing antacids can cause diarrhea, and
should be used with caution in older persons with renal
Erosive (hemorrhagic) gastritis can be caused by ingestion of
dysfunction. Aluminum-containing antacids can cause constipation,
substances that irritate the gastric mucosa. Fundic gland gastritis
osteomalacia, and hypophosphatemia. Cimetidine has the greatest
(type A) is associated with diffuse severe mucosal atrophy and the
chance for adverse reactions, including erectile dysfunction,
presence of pernicious anemia. Antral gland gastritis (type B) is
Gynecomastia, and confusion. Misoprostol's major side effects are
the most common form of gastritis, and is associated with
diarrhea and abdominal pain.
Helicobacter pylori and duodenal ulcers.

The surgical procedure of choice for older


Which of the following statements is
patients with GERD and Barrett's
accurate about patients diagnosed with
esophagus that is not reversible with
Zollinger-Ellison syndrome?
medical management is: Peptic ulcer occurs in 95% of patients
Esophagectomy. diagnosed with Zollinger-Ellison syndrome.
Total laryngectomy. Zollinger-Ellison syndrome is characterized by
gastric hyposecretion, because of dysfunction
Nissen's fundoplication. of the parietal cells.
Patients with Zollinger-Ellison syndrome with
Labyrinthectomy.
persistent symptoms that do not respond to
drug treatment should be referred to a
radiologist.
Treatment of choice for Zollinger-Ellison
In planning care for a patient with
syndrome is a cholecystectomy.
ulcerative colitis, the nurse should
Peptic ulcers occur in 95% of patients with Zollinger-Ellison anticipate which of the following
syndrome. Treatment might include tumor removal and surgical diagnostic procedures?
resection for older persons without surgical risk. Persons with Sigmoidoscopy
Zollinger-Ellison syndrome have gastric hypersecretion, and are
referred to a gastroenterologist if symptoms persist. Colonoscopy

Rectal mucosa biopsy

Which of these agents is a major All of the above


contributing factor in the promotion of
peptic ulcer disorder? Diagnosis of ulcerative colitis is confirmed with the use of
Candida albicans sigmoidoscopy, colonoscopy, and rectal mucosa biopsy.

Staphylococcus infection

Streptococcus infection The most common surgical procedures for


patients with ulcerative colitis are:
Helicobacter pylori infection Subtotal colectomy and ileostomy.

Colostomy and ileo-conduit.


Recurrence of peptic ulcers is related to Helicobacter pylori, use of
NSAIDs, smoking, and continued acid hypersecretion. Laparoscopic gastrectomy.

Segmental resection or colostomy.


Which of the following is not an indication
of diverticulitis? Surgery might be necessary for functional older patients with acute
Fever disease when drug therapy fails and when multiple precancerous
lesions are detected. The most common surgical procedures are
Leukocytosis subtotal colectomy and ileostomy.
Pain

Diarrhea Older patients with longstanding or severe


Crohn's disease can exhibit which of the
Diverticulitis is an infection from colonic diverticula. Fever, following?
leukocytosis, pain, and/or abdominal tenderness might be Hyperalbuminemia
indicators of diverticulitis.
Hypoalbuminemia
Decreased sedimentation rate

Nausea and vomiting

Age-related changes that affect the hematologic


system include: (Select all that apply.)
Correct Answers: Bone marrow in the long bones
decline.

Lymphocyte function, especially cellular immunity,


decreases.

Rationale: At about age 70, the amount of


bone marrow in the long bones declines steadily;
the number of stem cells in the marrow decreases;
and lymphocyte function, especially cellular
immunity, declines.

2. Anemia or insufficient hemoglobin content is


common in older persons. The client's body
compensates for the deficiency by:

Rationale: All anemias result in a loss of


oxygen-carrying capacity of the blood, and produce
generalized hypoxia. The body tries to compensate
by raising the heart and respiratory rates, shunting
blood to vital organs away from the skin, and
increasing blood viscosity in order to supply oxygen
to hypoxic tissues.

3. Which of the following statements accurately


describes normocytic anemia? Normocytic anemia
usually is caused by: sometimes is used to treat anemia and fatigue in
Your Answer: Concurrent chronic illness, such some older clients with non-myeloid cancers.
as chronic heart disease.
6. The most common cause of macrocytic
Rationale: Normocytic anemia usually is anemia in the older person is B12 or folate
caused by concurrent chronic illness such as chronic deficiency. Failure to absorb vitamin B12 from the
heart, respiratory, or renal disease or malignancy. G.I. tract is called:
Hemolytic anemia also is a normocytic anemia.
Correct Answer: Pernicious anemia.
4. The usual treatment for iron-deficiency
anemia includes: Rationale: Failure to absorb vitamin B12 from
Your Answer: Non-enteric-coated ferrous the G.I. tract is called pernicious anemia, and results
sulfate. from a lack of intrinsic factor. About 12% of older
people have low serum vitamin B12 levels.
Rationale: Once the cause of iron deficiency
has been identified, oral iron therapy is the 7. An elderly woman is seen at the outclient
preferred treatment. The usual treatment is ferrous clinic by a nurse practitioner for a routine
sulfate 325 mg p.o. daily. Enteric-coated and hematologic evaluation. Which of the following
sustained release formulations should be avoided, findings would alert the nurse practitioner of the
as they are poorly absorbed. need for a pending diagnosis of hemolytic anemia?
Correct Answer: Ongoing use of cephalosporin,
________________________________________ quinidine, triamterene, and aspirin
5. Erythropoietin sometimes is administered
subcutaneously to treat which of the following? Rationale: Drugs capable of causing
(Select all that apply.) hemolytic anemia include ibuprofen, L-dopa,
penicillin, cephalosporins, tetracycline,
Correct Answer: Clients with anemia and fatigue acetaminophen, aspirin, erythromycin, hydralazine,
related to non-myeloid cancers insulin, isoniazid, procainamide, quinidine, rifampin,
streptomycin, sulfonamides, methadone,
Rationale: Cancer in the late stages nearly hydrochlorothiazide, triamterene, and phenacetin.
always is accompanied by anemia of chronic
disease. Erythropoietin, which stimulates red blood 8. The nurse has provided nutritional teaching
cell production, is injected subcutaneously, and on foods high in folate to a client with folate
deficiency related to malabsorption syndromes and diagnose thrombocytopenia, the physician most
poor nutrition. Which of the following foods, if likely will order:
chosen by the client, indicates that the client
understands the teaching? Correct Answer: Bone marrow aspiration.
Correct Answer: Liver and dark green leafy
vegetables Rationale: Thrombocytopenia is characterized
by an increased number of circulating platelets in
Rationale: Foods high in folate are liver, the blood. Accurate diagnosis requires bone marrow
orange juice, cereals, whole grains, beans, nuts, and aspiration. Allogenic bone marrow transplantation is
dark leafy vegetables like spinach. prescribed for younger persons with myelofibrosis. A
splenectomy may be prescribed for persons with
9. Because older persons can have severe myelofibrosis.
anemia for a long period of time without detection,
when diagnosed, quick reversal is warranted. Which 11. When caring for a client with a diagnosis of
of the following orders most likely would be thrombocytopenia, the nurse should plan to:
prescribed at this time? Your Answer: Avoid invasive procedures.
Your Answer: Ferrous sulfate 325 mg orally
three times a day Rationale: Thrombocytopenia is characterized
Correct Answer: Packed red blood cells followed by an increased number of circulating platelets in
by oral furosemide (Lasix) the blood. Older persons with thrombocytopenia are
at significantly increased risk of thrombosis, and
Rationale: Older persons might have heart careful monitoring of platelet levels and symptoms
problem that are compounded by severe anemia. is indicated.
The physician can prescribe blood transfusions to 12. In formulating a nursing diagnosis of risk for
reverse the severity of the anemia, and a diuretic infection for a client with chronic lymphoid leukemia
such as furosemide (Lasix) orally between units to (CLL), nursing measures should include: (Select all
prevent fluid overload and the development of that apply.)
congestive heart failure (CHF). Correct Answers: Placing the client in protective
isolation.
10. An elderly client is admitted to the hospital
Emergency Department (ED) with complaints of Limiting visitors who have colds and infections.
headache, visual disturbances, and burning pain,
and erythema of the hands and feet. To accurately Ensuring meticulous handwashing by all persons
coming in contact with the client. Your Answer: "It only hurts when I move. I
don't need any pain medication."
Rationale: Chronic lymphoid leukemia (CLL) is
characterized by a proliferation and accumulation of Rationale: Postoperative and chronic pain
small, abnormal mature lymphocytes in bone should be carefully assessed and treated to prevent
marrow, peripheral blood, and body tissues. older clients from the hazards of immobility. The
Infections and fever are frequent complications of older client who says, "It only hurts when I move. I
CLL. don't need any pain medication," is at risk for
immobility and DVT formation.
13. Which question by the nurse would be
beneficial in assessing the needs of a client with Which of the following is the major mechanism for
multiple myeloma? maintaining calcium balance in the body?
Correct Answer: Bone remodeling
Correct Answer: "Describe your pain."
Rationale: Bone accounts for roughly 99% of
Rationale: Multiple myeloma is a malignancy calcium in the body. Throughout life, new bone
that results from the overproduction and continually is deposited and reabsorbed in response
accumulation of immature plasma cells in the bone to hormonal, dietary, and mechanical stimuli. When
marrow. Bone pain in the lower back or ribs is the put together, these processes are called bone
most common early symptom of multiple myeloma. remodeling.
Body alignment does not directly affect calcium
14. Which of the following manifestations would balance. Exercise, particularly weight-bearing
be directly associated with Hodgkin's disease? exercise, is important for bone health.

Correct Answer: Painless, enlarged lymph nodes

Rationale: Hodgkin's disease usually presents


as painless enlarged lymph nodes. The diagnosis is 2. Osteoporosis can be defined as:
made by lymph node biopsy. Your Answer: Loss of bone density.

15. Which of the following statements made by Rationale: Osteoporosis is loss of bone
the older client indicates risk for immobility and density. Specific causes in the elderly include
deep vein thrombosis (DVT) formation? Vitamin D deficiencies and the use of glucocorticoid
drugs. Phagocytic activity is not a contributing Rationale: Immediate interventions such as
factor here. airway management and vital signs are priority. The
patient with the fractured ankle is requesting pain
3. Which of the following risk factors are medication, but the type of pain is not indicated.
directly related to common musculoskeletal The client with the wet dressing is likely the lowest-
problems in older persons? (Select all that apply.) priority client.
Correct Answers: Phosphate depletion
6. The physician prescribes alendronate
Vertebral collapse (Fosamax) and risedronate (Actonel) for a 65-year-
old female for which of the following purposes?
Alteration in vitamin D metabolism
Correct Answer: Inhibiting osteoclastic activity,
Osteoid bone is produced in excess of and decreasing the incidence of vertebral and non-
mineralization. vertebral fractures

4. Which of the following nursing diagnoses Rationale: Fosamax and Actonel are potent
would be assigned priority for the older person with drugs that inhibit osteoclastic activity and help to
common musculoskeletal problems? decrease the incidence of vertebral and non-
Correct Answer: Mobility impairment, physical: vertebral fractures in post-menopausal women. The
Related to pain or discomfort actions of these drugs do not include preventing
thrombus formation, binding bile acids, or acting as
Rationale: Musculoskeletal status - including an anti-inflammatory agent.
coordination, gait, muscle size and strength, muscle
tone, range of motion, and functional mobility - will 7. Which of the following nursing implications
cause altered mobility or impairment in mobility for are important when Fosamax and Actonel are
patients with common musculoskeletal problems. prescribed for an older patient? (Select all that
apply.)
5. Which of the following patients with Correct Answers: Take these drugs on an empty
musculoskeletal problems should the nurse observe stomach, first thing in the morning, with water.
first?
Correct Answer: A client who has just returned Remain upright for 30 minutes and do not eat or
from the operating room (OR) drink anything else for 30 minutes.
Rationale: Fosamax causes esophagitis if predisposition is a major factor in both the
there is prolonged contact with the esophageal susceptibility and the severity of RA.
mucosa, or if it does not pass through the
esophagus completely. Therefore, Fosamax must be 10. The focus of treatment for rheumatoid
taken with a sufficient amount of water. To promote arthritis is related directly to joint damage that is
complete passage of the drug, it should be taken in caused by:
an upright position, to prevent delay of the drug
through the esophagus. Correct Answer: Pannus.

8. A patient is diagnosed with osteomalacia. Rationale: Pannus is vascular granulation


The physician prescribes vitamin D replacement tissue composed of inflammatory cells that erodes
50,000 U/day for two weeks, to be followed by a articular cartilage and eventually destroys bones. A
daily dose of 800 U/day. The primary goal for this vasculitis usually indicates blood vessel
pharmacological treatment is that: involvement. Sjögren’s syndrome is due to
Your Answer: The patient will achieve effective inflammatory cells and immune complexes
remineralization of bone structure. obstructing secretory ducts and glands. Felty's
syndrome is characterized by rheumatoid arthritis,
Rationale: The goal of pharmacological hepatosplenomegaly, and leukopenia.
treatment for osteomalacia is to remineralize the
bone. All older persons with osteomalacia need to 11. The definitive finding for persons with gout
have adequate vitamin D intake to help alleviate is:
calcium deficiency.
Correct Answer: Urate crystals in synovial fluid.
9. Which of the following is a major factor in the
susceptibility and severity of rheumatoid arthritis Rationale: Gout results from the deposit of
(RA)? urate crystals in a peripheral joint. The definitive
Correct Answer: Genetic predisposition finding for a diagnosis of gout is urate crystals in the
synovial fluid, usually via an arthrocentesis.
Rationale: The cause of RA is unknown, but it Hyperuricemia is excess uric acid in the blood.
most likely is due to a variety of unknown Diffuse periarticular erythema is not definitive for
environmental factors, including infectious agents gout.
and chemical exposures, all triggering an
autoimmune response. However, a person's genetic 12. A patient with pseudogout most likely would
complain of pain in what specific area of the body? Correct Answers: Promoting a diet with adequate
Your Answer: Knee calcium and vitamin D

Rationale: The knee joint is the most common Encouraging weight-bearing exercise
joint affected for most people. Other joints that
might be affected include the shoulder, hip, and Emphasizing the importance of smoking cessation
elbow.
Rationale: Osteoarthritis is the most common
13. Which of the following statements is form of arthritis in the United States, more women
accurate about hip fractures? are affected than are men? Recent studies have
determined that aging alone does not cause the
Correct Answer: Extracapsular fractures cause disease; therefore, any woman, by the age of 40,
acute blood loss from the vascular cancellous bone might show evidence of primary osteoarthritis on x-
surfaces, but rarely cause avascular necrosis. ray.

Rationale: Extracapsular fractures cause Which of the following persons would most likely be
acute blood loss from the vascular cancellous bone diagnosed with diabetes mellitus? A 44-year-old:
surfaces. Correct Answer: African-American woman.

14. Which of these findings correlates best with Rationale: Age-specific prevalence of
a diagnosis of Paget's disease? diagnosed diabetes mellitus (DM) is higher for
Correct Answer: Complaints of bone pain African-Americans and Hispanics than for
Caucasians. Among those younger than 75, black
Rationale: Bone pain is the most frequently women had the highest incidence.
reported symptom, described as deep and aching.
Hyperuricemia is present, and the skin usually is 2. Which of the following factors are risks for
flushed and warm. Nausea and vomiting are a the development of diabetes mellitus? (Select all
nonspecific presentation. that apply.)
Correct Answers: Age over 45 years
15. Principles related to nursing management of
older persons with arthritis include: (Select all that Overweight with a waist/hip ratio >1
apply.)
Maintaining a sedentary lifestyle
Nursing Process: Assessment
Rationale: Aging results in reduced ability of Client Need: Health Promotion and Maintenance
beta cells to respond with insulin effectively. Taxonomic Level: Application
Overweight with waist/hip ratio increase is part of ________________________________________
the metabolic syndrome of DM II. There is an 5.
increase in atherosclerosis with DM due to the When an older adult is admitted to the hospital with
metabolic syndrome and sedentary lifestyle. a diagnosis of diabetes mellitus and complaints of
rapid-onset weight loss, elevated blood glucose
3. Which laboratory test should a nurse levels, and polyphagia, the gerontology nurse
anticipate a physician would order when an older should anticipate which of the following secondary
person is identified as high-risk for diabetes medical diagnoses?
mellitus? (Select all that apply.) Your Answer: Pituitary tumor
Correct Answer: Pancreatic tumor
Correct Answers: Fasting Plasma Glucose (FPG)
Rationale: The onset of hyperglycemia in the
Two-hour Oral Glucose Tolerance Test (OGTT) older adult can occur more slowly. When the older
adult reports rapid-onset weight loss, elevated blood
Rationale: When an older person is identified glucose levels, and polyphagia, the healthcare
as high-risk for diabetes, appropriate testing would provider should consider pancreatic tumor.
include FPG and OGTT. A FPG greater than 140 Nursing Process: Assessment
mg/dL usually indicates diabetes. The OGTT is to Client Need: Physiologic Integrity, Reduction of Risk
determine how the body responds to the ingestion Potential
of carbohydrates in a meal. HbA1C evaluates long- Taxonomic Level: Evaluation
term glucose control. A finger stick glucose three ________________________________________
times daily spot-checks blood glucose levels. 6.
The principal goals of therapy for older patients who
4. Of which of the following symptoms might an have poor glycemic control are:
older woman with diabetes mellitus complain? Your Answer: Improving self-care through
Your Answer: Pain intolerance education.
Correct Answer: Perineal itching Correct Answer: All of the above.

Rationale: Older women might complain of Rationale: The principal goals of therapy for
perineal itching due to vaginal candidiasis. older persons with diabetes mellitus and poor
glycemic control are enhancing quality of life, Correct Answer: Somogyi phenomenon
decreasing the chance of complications, improving
self-care through education, and maintaining or Rationale: Somogyi phenomenon manifests
improving general health status. itself with nocturnal hypoglycemia, followed by a
Nursing Process: Planning marked increase in glucose and increase in ketones.
Client Need: Safe, Effective Care Environment
Taxonomic Level: Comprehension Nursing Process: Analysis
________________________________________ Client Need: Physiologic Integrity
7. Taxonomic Level: Analysis
Which of the following is accurate pertaining to ________________________________________
physical exercise and type 1 diabetes mellitus? 9.
Your Answer: Strenuous exercise is beneficial The primary purpose for sulfonylureas, such as long-
when the blood glucose is high. acting glyburide (Micronase), is to:
Correct Answer: Physical exercise can slow the Your Answer: Improve insulin sensitivity and
progression of diabetes mellitus. decrease hyperglycemia.
Correct Answer: Stimulate the beta cells of the
Rationale: Physical exercise slows the pancreas to secrete insulin.
progression of diabetes mellitus, because exercise
has beneficial effects on carbohydrate metabolism Rationale: Sulfonylureas such as glyburide
and insulin sensitivity. Strenuous exercise can cause are used only with patients who have some
retinal damage, and can cause hypoglycemia. remaining pancreatic-beta cell function. These
Insulin and foods both must be adjusted to allow drugs stimulate insulin secretion, which reduces
safe participation in exercise. liver glucose output and increases cell uptake of
Nursing Process: Application glucose, enhancing the number of and sensitivity of
Client Need: Physiologic Integrity cell receptor sites for interaction with insulin.
Taxonomic Level: Analysis Nursing Process: Analysis
________________________________________ Client Need: Physiologic Integrity, Pharmacologic
8. Therapy
A diabetic patient experiencing a reaction of Taxonomic Level: Knowledge
alternating periods of nocturnal hypoglycemia and ________________________________________
hyperglycemia might be manifesting which of the 10.
following? One of the benefits of Glargine (Lantus) insulin is its
Your Answer: Brittle diabetes ability to:
Your Answer: Simplify the dosing and better Client Need: Physiologic Integrity
control blood glucose levels during the day. Taxonomic Level: Analysis
Correct Answer: Release insulin evenly ________________________________________
throughout the day and control basal glucose levels. 12.
Which of the following diagnostic studies is done to
Rationale: Glargine (Lantus) insulin is determine the size and composition of the thyroid
designed to release insulin evenly throughout the gland?
day and control basal glucose levels. Your Answer: Thyroid scan with RAI 123I
Nursing Process: Evaluation Correct Answer: Ultrasonography
Client Need: Physiologic Integrity, Pharmacologic
Therapy Rationale: Although thyroid scans frequently
Taxonomic Level: Evaluation are done to evaluate the thyroid gland, I 123 is used
________________________________________ to destroy overactive thyroid cells such as are seen
11. in thyroid cancer. Ultrasonography can be used
A frail elderly patient with a diagnosis of type 2 early in the evaluation process to rule out Graves'
diabetes mellitus has been ill with pneumonia. The disease, nodular goiter, or other thyroid dysfunction.
client's intake has been very poor, and she is Nursing Process: Assessment
admitted to the hospital for observation and Client Need: Physiologic Integrity, Reduction of Risk
management as needed. What is the most likely Potential
problem with this patient? Taxonomic Level: Knowledge
Your Answer: Diabetic ketoacidosis is ________________________________________
occurring. 13.
Correct Answer: Hyperglycemic hyperosmolar Which of the following is a drug of choice for
non-ketotic coma hyperthyroidism (Graves' disease)?
Your Answer: Digoxin (Lanoxin)
Rationale: Illness, especially with the frail Correct Answer: Propylthiouracil (PTU)
elderly patient whose appetite is poor, can result in
dehydration and HHNC. Insulin resistance usually is Rationale: Propylthiouracil (PTU) initially is
indicated by a daily insulin requirement of 200 units given in divided doses, and functions to block
or more. Diabetic ketoacidosis, an acute metabolic thyroid hormone synthesis.
condition, usually is caused by absent or markedly Nursing Process: Assessment
decreased amounts of insulin. Client Need: Physiologic Integrity, Pharmacologic
Nursing Process: Analysis Therapy
Taxonomic Level: Knowledge Client Need: Physiologic Integrity, Pharmacologic
________________________________________ Therapy
14. Taxonomic Level: Knowledge
The nurse is caring for a patient with a diagnosis of
hypothyroidism. Which nursing diagnosis should the Which of the following are least likely to be early
nurse most seriously consider when analyzing the signs of cardiac problems in older persons? (Select
needs of the patient? all that apply.)
Your Answer: Diarrhea related to increased Your Answer: Mental status changes
peristalsis Correct Answers: Mental status changes
Correct Answer: Hypothermia related to slowed
metabolic rate Agitation

Rationale: Thyroid hormone deficiency results Frequent falls


in reduction in the metabolic rate, resulting in
hypothermia, and does predispose the older adult to Rationale: Many cardiovascular functions are
a host of other health-related issues. One quarter of complicated in that they involve many other
affected elderly experience constipation. systems. Mental status changes, agitation, and falls
Nursing Process: Planning can be early signs of cardiac problems in the older
Client Need: Physiologic Integrity, Reduction of Risk person. Changes in function in the GI system are not
Potential typical signs of a cardiac problem.
Taxonomic Level: Analysis Integrated Process: Nursing Process; Evaluation
________________________________________ Cognitive Level: Analysis
15. NCLEX-RN Test Plan: Physiological Integrity;
Which of the following medications are most likely Reduction of Risk
to cause hypothyroidism? (Select all that apply.) ________________________________________
Your Answer: Acetylsalicylic acid (aspirin) 2.
Correct Answers: Acetylsalicylic acid (aspirin) A patient has been diagnosed with Right-Sided
Congestive Heart Failure, and is confused about
Furosemide (Lasix) return of deoxygenated blood from the tissue. To
clarify the confusion, which chamber of the heart
Rifampin (Rifadin) receives blood from systemic circulation?
Your Answer: Right atrium
Nursing Process: Assessment
Rationale: The right atrium is a thin-walled Nurses can best help older clients prevent
structure that receives deoxygenated blood from all hypertension by teaching:
the peripheral tissues by way of the superior and Your Answer: Low-fat, low-cholesterol diets.
inferior vena cava and from the heart muscle by Correct Answer: How to maintain a normal blood
way of the coronary sinus. pressure.
Integrated Process: Teaching Learning
Cognitive Level: Knowledge Rationale: Hypertension is a major risk factor
NCLEX-RN Test Plan: Health Promotion and for other cardiovascular conditions. In persons older
Maintenance than 50, systolic blood pressure greater than 140
________________________________________ mm Hg is a much more important cardiovascular
3. disease risk factor than is diastolic blood pressure.
It is important that the nurse be knowledgeable The risk of cardiovascular disease, beginning at
about cardiac output in order to: 115/75 mm Hg, doubles with each increment of
Your Answer: Determine the electrical activity 20/10 mm Hg. Answers 1, 2, and 3 are important
of the myocardium. elements to include in education of a patient with
Correct Answer: Evaluate blood flow to peripheral blood pressure elevation, and are included in the
tissues. correct answer.
Integrated Process: Nursing Process;
Rationale: Blood flow to the tissues is Implementation
measured clinically as the cardiac output, and Cognitive Level: Evaluation
assists to predict tissue perfusion. Electrical activity NCLEX-RN Test Plan: Health Maintenance and
is evaluated more effectively by EKG. While the Promotion
cardiac output is important for perfusion and ________________________________________
oxygenation of tissues, the oxygen saturation would 5.
provide more valuable information. Nutritional Modification of lifestyle behaviors to help manage
changes would be targeted to sodium and would hypertension does not include which of the
depend on symptoms of disease. following? (Select all that apply.)
Integrated Process: Nursing Process; Planning Your Answer: The DASH diet
Cognitive Level: Evaluation Correct Answer: Alcohol intake with meals
NCLEX-RN Test Plan: Health Promotion and
Maintenance Rationale: Weight loss of even 10 pounds;
________________________________________ fruits, vegetables, and whole grains; the DASH diet;
4. and a daily exercise regimen will help reduce high
blood pressure. Alcohol intake with meals should be Pulmonary embolus
reduced to help manage high blood pressure.
Integrated Process: Nursing Process; Planning Rationale: Herpes zoster (shingles) manifests
Cognitive Level: Synthesis as a vesicular rash along a dermatome, not chronic
NCLEX-RN Test Plan: Health Maintenance and ischemic pain.
Promotion Integrated Process: Nursing Process; Evaluation
________________________________________ Cognitive Level: Evaluation
6. NCLEX-RN Test Plan: Physiological Integrity;
A patient receiving the drug simvastatin (Zocor) Physiological Adaptation
should be taught this medication helps to prevent ________________________________________
coronary heart disease by: 8.
Your Answer: Increasing triglycerides. Older clients experiencing anginal pain with
Correct Answer: Controlling lower-density complaints of fatigue or weakness usually are
lipoprotein. medicated with which of the following types of
medication?
Rationale: The Heart Protective Study has Your Answer: Sublingual nitroglycerin
also shown that controlling low-density lipoprotein
(LDL) with simvastatin (Zocor) assists in the Rationale: Angina frequently is managed with
prevention of coronary heart disease by raising HDL. sublingual nitroglycerin, which causes vasodilation
Responses 1, 3, and 4 are incorrect. and increases blood flow to the coronary arteries.
Integrated Process: Teaching Learning Cardiac glycosides are used to treat heart failure,
Cognitive Level: Analysis and morphine is used to treat myocardial infarction.
NCLEX-RN Test Plan: Physiological Integrity: The HMG-CoA reductase inhibitors are used for
Pharmacological and Parenteral Therapies patients with type 2 diabetes mellitus.
________________________________________ Integrated Process: Nursing Process; Analysis
7. Cognitive Level: Analysis
Which of the following are not direct causes of NCLEX-RN Test Plan: Physiological Integrity;
chronic ischemic pain? (Select all that apply.) Pharmacological Therapy
Your Answer: Acid reflux ________________________________________
Correct Answers: Aortic stenosis 9.
Which of the following diagnostic studies most likely
Acid reflux would confirm a myocardial infarction?
Your Answer: Creatinine kinase (CK)
Correct Answer: Troponin T levels Integrated Process: Nursing Process; Assessment
Cognitive Level: Analysis
Rationale: CK-MB elevates 4-6 hours after NCLEX-RN: Physiological Integrity; Pharmacological
tissue necrosis. Troponin levels rise 6-8 hours after Therapy
the infarct (tissue necrosis) but also can occur with ________________________________________
other types of tissue damage. Myoglobin also 11.
elevates, but to a lesser degree. WBC levels elevate Which of the following diagnostic tests is preferred
with an inflammatory response. Troponin levels are for evaluating heart valve function?
more elevated than are the other cardiac enzymes, Your Answer: Echocardiogram
are more specific to cardiac tissue, and rise 6-8
hours after the infarct (tissue necrosis). Rationale: The echocardiogram is the
Integrated Process: Nursing Process: Analysis preferred test to evaluate heart valves, because it
Cognitive Level: Analysis allows the visualization of the valves as they open
NCLEX-RN Test Plan: Physiological Integrity; and close. A chest x-ray will determine the size of
Reduction of Risk Potential the heart, the duplex measures blood flow through
________________________________________ major arteries, and an electrocardiogram identifies
10. electrical activity.
A client with post-myocardial infarction develops Integrated Process: Nursing Process; Analysis
acute bacterial pericarditis. Which of the following Cognitive Level: Analysis
medications would the physician most likely NCLEX-RN Test Plan: Physiological Integrity;
prescribe as the primary drug? Reduction of Risk Potential
Your Answer: Acetaminophen (Tylenol) ________________________________________
Correct Answer: Ticarcillin disodium (Ticar) 12.
An elderly client is being monitored for evidence of
Rationale: Acute bacterial pericarditis is a congestive heart failure. To detect early signs of
complication that can occur post-myocardial heart failure, the nurse would instruct the certified
infarction. Acute bacterial pericarditis usually nursing attendant (CNA) to do which of the following
requires antibiotics. NSAIDs usually are prescribed during care of the patient?
to relieve pain from the inflammatory process. If the Your Answer: Observe electrocardiogram
NSAIDs do not relieve pain within 48 to 96 hours, readings and report deviations to the nurse.
corticosteroids are ordered. There is no mention of Correct Answer: Accurately weigh the patient,
pain in the stem of the question. Trisoralen is used and report and record the readings.
to repigment skin for persons with vitiligo.
Rationale: Due to fluid accumulation, an and suppresses impulse conduction through the AV
expanded blood volume can result when the heart node, all of which slows the progression of the
fails. Body weight is a sensitive indicator of water disease process. Carvedilol (Coreg) is another beta-
and sodium retention, which will manifest itself with adrenergic blocker used to treat heart failure.
edema, dyspnea - especially nocturnal - and pedal Integrated Process: Nursing Process; Assessment
edema. Patients also should be instructed about the Cognitive Level: Analysis
need to perform daily weights upon discharge to NCLEX-RN Test Plan: Physiological Integrity;
monitor body water. It is not within the role of the Pharmacological Therapy
CNA to monitor ECG readings, and ambulation is not ________________________________________
an assessment. Vital signs every 15 minute are not 14.
necessary for this level of patient care. A common arrhythmia found in some older clients is
Integrated Process: Communication and chronic atrial fibrillation. Based on the nurse's
Documentation knowledge of the disease pathology, which of the
Cognitive Level: Planning following prescriptions should the nurse expect to
NCLEX-RN Test Plan: Safe, Effective Care be ordered?
Environment Your Answer: Warfarin sodium (Coumadin)
________________________________________
13. Rationale: Chronic atrial fibrillation places a
Which of the following drug classifications should patient at high risk for clot formation. Warfarin
the nurse question if prescribed for a person with sodium frequently is ordered as an anti-coagulant.
congested heart failure (CHF)? Aspirin will not prevent clots associated with atrial
Your Answer: Beta-adrenergic blocker fibrillation. Zocor is used to lower LDL and increase
Correct Answer: Rosiglitazone (Avandia) HDL. Navelbine is an anti-neoplastic.
Integrated Process: Nursing Process; Analysis
Correct answer: Thiazolidinediones, like Cognitive Level: Analysis
rosiglitazone (Avandia), are glucose-reducing drugs NCLEX-RN Test Plan: Physiological Integrity;
that are prescribed for persons with type 2 diabetes Pharmacological Therapy
mellitus. ACE inhibitors, such as Lisinopril, are first- ________________________________________
line drugs used to treat CHF. Propranolol (Inderal), a 15.
beta blocker, has remained one of the most widely Which of the following assessments would be an
used beta-blocking drugs. It blocks both beta1 and important finding for a patient with arterial disease?
beta2 receptors in various organs, resulting in Your Answer: Intermittent claudication with
reduction of heart rate and the force of contraction, exercise
decreased alveolar surface, and increased chest
Rationale: Intermittent claudication is a wall stiffness. Bronchovesicular breath sounds are
common finding in persons with arterial disease, heard over major bronchi where fewer alveoli are
usually due to progression of atherosclerosis and located.
alteration of tissue perfusion to the extremities. In Integrated Process: Nursing Process; Assessment
venous disease, valves of the veins in the Cognitive Level: Analysis
extremities become incompetent, resulting in higher NCLEX-RN Test Plan: Health Promotion and
pressures than normal in the veins. The pressure is Maintenance
transmitted to the capillaries of the lower ________________________________________
extremities, resulting in thickening and non-pitting 2.
edema of tissues around the ankles. Prolonged An older client is receiving postural drainage
thickening results in the red blood cells' being treatments but is unable to expel the secretions.
pressed outside the capillaries. The cells eventually The client is confused, and having difficulty
break down, resulting in collection of hemosiderin following instructions. The best response by the
deposits being collected in the area. Altered nurse would be to:
sensation to touch would be due to neuropathic Your Answer: Administer humidified oxygen.
changes commonly found with diabetes mellitus. Correct Answer: Suction out the secretions.
Integrated Process: Nursing Process: Analysis
Cognitive Level: Analysis Rationale: Postural drainage loosens
NCLEX-RN Test Plan: Physiological Integrity; secretions, which patients usually expectorate when
Reduction of Risk Potential loosened. Since the client is confused, and cannot
Which of the following is a normal finding when follow instructions, the nurse must suction the
assessing the respiratory system of an elderly secretions to enhance effective breathing.
client? Integrated Process: Nursing Process;
Your Answer: A decreased anteroposterior Implementation
diameter and increased alveolar surface Cognitive Level: Synthesis
Correct Answer: Decreased mobility of the thorax NCLEX-RN Test Plan: Physiological Integrity:
and increased chest wall stiffness Physiological Adaptation
________________________________________
Rationale: The normal aging process is 3.
accompanied by physiologic changes to the Chest percussion has been ordered for a client on
respiratory system such as stiffening of elastin and bedrest with respiratory infections and increased
collagen tissues, increased alveolar diameter, secretions. The nurse should use which of the
following hand positions to administer chest ________________________________________
percussion? 5.
Your Answer: Cupped-hand position The nurse is teaching a group of older clients about
the functions of the respiratory tract as part of
Rationale: Cupped-hand position enhances health promotion. Which statement made by
vibration, which helps to loosen secretions, clear patients would indicate a need for further teaching?
airway, and improve gas exchange. The flat, fisted, Your Answer: "Parts of our respiratory system
and flexed hand positions can cause pain and harm help us produce speech and better communication."
the patient. Correct Answer: "The lungs keep our blood
Integrated Process: Nursing Process; pressure in normal range, so we can breathe
Implementation better."
Cognitive Level: Analysis
NCLEX-RN Test Plan: Physiological Integrity; Rationale: The respiratory system performs
Reduction of Risk Potential several secondary functions, including maintenance
________________________________________ of acid-base balance, production of speech, and
4. maintenance of body water and heat balance. The
All of the following nursing diagnoses are important heart and cardiovascular system are responsible for
for a client with chronic pulmonary emphysema the blood pressure.
(COPD). Which would receive priority when planning Integrated Process: Teaching Learning
nursing interventions? Cognitive Level: Analysis
Your Answer: Activity intolerance NCLEX-RN Test Plan: Health Promotion and
Correct Answer: Impaired gas exchange Maintenance
________________________________________
Rationale: Physical assessment of older 6.
persons with COPD might find ineffective airway Which of the following diagnostic studies is most
clearance, but the classic physiology alteration is helpful in determining the presence of asthma?
the destruction of the alveoli with resulting Your Answer: Spirometry
complications. Therefore, impaired gas exchange Correct Answer: Pulmonary function tests (PFTs)
always is present.
Integrated Process: Nursing Process; Analysis Rationale: Pulmonary function tests are the
Cognitive Level: Analysis most reliable way to diagnose asthma and
NCLEX-RN Test Plan: Physiological Integrity; differentiate it from other illnesses like COPD. Large
Adaptation numbers of eosinophils indicate an allergic
component and a predictor of asthma. Spirometry is Pharmacological and Parenteral
used to measure the volume of air expired in one ________________________________________
second from maximum inspiration. Incentive 8.
spirometer is used for breathing exercises with Which of the following medications should the nurse
patients who have respiratory problems; pre- and anticipate the physician would prescribe for
postop patients also benefit from the use of the "rescue" from sudden-onset wheezing, tightness in
incentive spirometer. the chest, or shortness of breath due to an asthma
Integrated Process: Nursing Process; Assessment diagnosis?
Cognitive Level: Analysis Your Answer: (blank)
NCLEX-RN Test Plan: Physiological Integrity;
Reduction of Risk Potential ________________________________________
________________________________________ 9.
7. Which of the following interventions would be a
Corticosteroid therapy is the most effective anti- priority in the plan of care for the person with
inflammatory treatment for asthma. pulmonary emphysema? (Select all that apply.)
The nurse administering oral corticosteroids should Your Answer: Maintaining hydration status
observe the client for adverse effects of: (Select all Correct Answer: Low-flow oxygen via face mask
that apply.)
Your Answer: Hyponatremia. Rationale: Patients with pulmonary
Correct Answers: Increase in intraocular pressure. emphysema have a low pO2 that keeps them alive
(their hypoxic drive). High levels of oxygen would
Pathologic fractures. diminish their response to breathe. Persons with
pulmonary emphysema have narrowed airways.
Rationale: Corticosteroids accelerate Aminophylline is a bronchodilator that works to
osteoporosis and increase the risk for pathologic open narrowed airways and improve airflow and gas
fractures. Corticosteroids result in sodium retention exchange. Patients with pulmonary emphysema
(not hyponatremia), which causes fluid retention, often have respiratory infection, and are treated
which could result in increased intraocular pressure. with broad-spectrum antibiotics, like ampicillin.
Weight gain, not weight loss, is a side effect of These patients often are dehydrated, and need
corticosteroid therapy. secretions liquefied for easier expectoration.
Integrated Process: Nursing Process; Assessment Limiting fluids would help decrease airway
Cognitive Level: Evaluation clearance and therefore diminish gas exchange.
NCLEX-RN Test Plan: Physiological Integrity; Integrated Process: Nursing Process; Analysis
Cognitive Level: Analysis things as well. An effective coping strategy is
NCLEX-RN Test Plan: Physiological Integrity; drawing strength from adversity.
Physiological Adaptation Integrated Process: Caring
________________________________________ Cognitive Level: Synthesis
10. NCLEX-RN Test Plan
A patient has been instructed in pursed-lip ________________________________________
breathing. The patient asks the nurse the purpose 12.
behind this breathing pattern. The nurse's best Which of the following measures is the most
response to this question would be: important action in preventing the transmission of
Your Answer: "Pursed-lip breathing prevents pulmonary tuberculosis?
airway collapse and enhances effective breathing." Your Answer: Adequate ventilation of rooms

Rationale: Pursed-lip breathing during Rationale: Pulmonary tuberculosis is an


exhalation is a useful technique for preventing early airborne disease that is spread from person to
airway collapse. person by droplets when an infected person coughs,
Integrated Process: Teaching/Learning sneezes, speaks, sings, or laughs. Responses 1, 2,
Cognitive Level: Analysis and 3 are not adequate interventions for the
NCLEX-RN Test Plan: Psychosocial Integrity prevention of tuberculosis. Having rooms with
________________________________________ adequate ventilation of non-recirculated air is the
11. best plan.
During an initial interview, the charge nurse informs Integrated Process: Nursing Process; Planning
a client that even though he is the youngest client Cognitive Level: Analysis
in the nursing home, he will be required to NCLEX-RN Test Plan: Health Promotion and
participate in activities with older patients. He tells Maintenance
the nurse that he can handle the interaction, but will ________________________________________
need the nurse's help during the initial adjustment. 13.
The responses from the patient indicate which of Clients identified with pulmonary tuberculosis due to
the following? M. tuberculosis usually are placed on Isoniazid (INH)
Your Answer: A stressful situation in which to prevent active disease. Client teaching prior to
challenge and gain are anticipated discharge from a clinic setting would place priority
on which of the following post-discharge
Rationale: Selye believes that it is not just the instructions?
"bad" things in life that cause stress, but the "good" Your Answer: Follow-up care will include
reading a PPD test within 72 hours. NCLEX-RN Test Plan: Physiological Integrity;
Correct Answer: Liver function tests should be Reduction of Risk Potential
monitored routinely. ________________________________________
15.
Rationale: Isoniazid causes hepatotoxicity; A patient is being treated for community-acquired
therefore, persons on Isoniazid should have liver pneumonia. Which of the following drugs should the
function tests routinely monitored. After diagnosis of nurse question, related to this treatment?
TB, a PPD always will be positive. Headache and Your Answer: Amoxicillin (Amoxil)
weight gain are not priorities. Correct Answer: Chlorzoxazone (Paraflex)
Integrated Process: Teaching Learning
Cognitive Level: Analysis Rationale: Chlorzoxazone is a skeletal muscle
NCLEX-RN Test Plan: Physiological Integrity; relaxant. Erythromycin, azithromycin, and
Pharmacological Therapies amoxicillin are oral antibiotics recommended to
________________________________________ treat older persons with community-acquired
14. pneumonia.
Patients with suspicious masses in the lung as seen Integrated Process: Nursing Process; Evaluation
on chest x-ray could have which of the following Cognitive Level: Analysis
diagnostic studies to provide information on soft NCLEX-RN Test Plan: Physiological Integrity;
tissue masses? Pharmacological and Parenteral Therapies
Your Answer: Pulmonary function tests (PFTs)
Correct Answer: CAT scan and magnetic
resonance imaging (MRI) One of the challenges in meeting the nutritional
needs of the elderly is that the elderly:
Rationale: The chest x-ray usually is the first Your Answer: Have a decreased need for
examination that the physician will order. Suspicious almost all nutrients.
masses seen on the chest x-ray could signal the Correct Answer: Have decreased caloric needs
need for CAT scan and an MRI to provide additional but constant or increased needs for vitamins and
information about soft tissue masses. The minerals.
gastrograffin swallow test would be used to
diagnose an esophageal rupture, while a wedge Rationale: A decreasing metabolic rate (or
resection would be treatment for a lung mass. resting energy expenditure) means fewer calories
Integrated Process: Nursing Process; Analysis are required, but the DRI for most nutrients remains
Cognitive Level: Analysis unchanged or might be increased in the elderly,
necessitating careful dietary planning to meet those the elderly to prevent constipation?
needs. Loss of interest in food might mean Your Answer: Drink coffee or tea only in the
physiologic problems exist. Dental problems for the morning.
elderly are not nearly as common today as they Correct Answer: Drink a glass of water several
were a generation ago. times a day.
Nursing Process: Planning
Cognitive Level: Analysis Rationale: Adequate fluid intake, along with
NCLEX-RN Test Plan: Physiological Integrity adequate fiber, are the best dietary measures to
________________________________________ prevent constipation. Daily stool softeners can lead
2. to dependency on them for bowel movements.
When an elderly person is hospitalized with Chewing is important, but will not prevent or control
pneumonia, what would be the best action by the constipation. Coffee or tea taken only once per day
nurse to encourage eating? will not supply enough fluid to prevent constipation.
Your Answer: Secure an order for a daily Nursing Process: Planning
multivitamin to stimulate appetite. Cognitive Level: Application
Correct Answer: Provide small, more frequent NCLEX-RN Test Plan: Physiological Integrity
meals. ________________________________________
4.
Rationale: An elderly person is more likely to What type of diet will the nurse recommend to most
eat if portions are small, more frequent, and healthy elderly clients?
nutrient-dense, as opposed to a full meal tray three Your Answer: A diet low in fat and protein
times a day. A vitamin will not stimulate appetite. Correct Answer: A diet high in complex
Snacks are not necessary if small, frequent meals carbohydrates and fiber
are provided. Hydration is always important, but
fluids do not provide the nutrients needed for Rationale: Complex carbohydrates and fiber
healing. provide some protein in addition to some necessary
Nursing Process: Implementation vitamins and minerals. They aid digestion, and have
Cognitive Level: Application a lower glycemic load. For an active, healthy elderly
NCLEX-RN Test Plan: Health Promotion and person, 1,200 calories per day will not be adequate
Maintenance to prevent weight loss. Inadequate protein does not
________________________________________ allow for tissue maintenance and repair. High-
3. carbohydrate diets do not provide adequate balance
What preventive measures might the nurse teach to caloric distribution.
Nursing Process: Planning unknown quality, and also might interact with
Cognitive Level: Application medications. Serious side effects are more likely
NCLEX-RN Test Plan: Health Promotion and when used with some prescription medications.
Maintenance Family and friends are not the best source of health
________________________________________ information.
5. Nursing Process: Evaluation
An elderly client has been admitted to a nursing Cognitive Level: Analysis
home, and the nurse completes an assessment. NCLEX-RN Test Plan: Health Promotion and
Which finding might lead the nurse to suspect a Maintenance
nutritional alteration? ________________________________________
Your Answer: Moist conjunctiva 7.
Correct Answer: Ridged nails The nurse who is preparing to insert a nasogastric
tube for enteral feedings recognizes that this
Rationale: Ridged, spoon-shaped nails are intervention is used for elderly clients who:
signs of long-term nutritional deficiencies. Shiny hair Your Answer: Cannot adequately metabolize
and moist conjunctiva are signs of nutritional nutrients.
balance. A pale tongue, in the absence of other Correct Answer: Are unable to ingest food.
pathology, does not signify nutritional inadequacy.
Nursing Process: Assessment Rationale: Enteral support is used for clients
Cognitive Level: Analysis who are unable to ingest adequate amounts of food
NCLEX-RN Test Plan: Physiological Integrity for tissue maintenance and repair. Digestion occurs
________________________________________ in the GI tract, and metabolism occurs at the cellular
6. level. The gag reflex is not related to nutritional
The elderly client who wants to take an herbal support unless it is present as a result of a medical
supplement for arthritis symptoms should be condition; a CVA, for example.
advised to: Nursing Process: Implementation
Your Answer: Verify their effectiveness with Cognitive Level: Analysis
friends or family members who have taken them. NCLEX-RN Test Plan: Physiological Integrity
Correct Answer: Consult their healthcare provider ________________________________________
about possible interactions with current 8.
medications. When working with elderly clients who require an
increased consumption of complete protein, the
Rationale: Herbal supplements might be of nurse recommends:
Your Answer: Iron-fortified cereal. Rationale: Dietary assessment and early
Correct Answer: 2% milk. intervention can improve overall quality of life and
possibly prevent disease or illness. Most acute
Rationale: After eggs and meats, milk is the conditions are not treated primarily by dietary
best source of complete protein. Legumes, cereal, intervention. Digestion occurs in the GI tract, and
and bread have some protein, but it is incomplete metabolism occurs at the cellular level.
protein. Nursing Process: Implementation
Nursing Process: Planning Cognitive Level: Analysis
Cognitive Level: Knowledge NCLEX-RN Test Plan: Health Promotion and
NCLEX-RN Test Plan: Health Promotion and Maintenance
Maintenance ________________________________________
________________________________________ 11.
9. Which nutrient is most likely to be deficient in
An appropriate dietary intervention for an elderly elderly residents of nursing homes?
client with dysphagia is: Your Answer: Protein
Your Answer: Thicken thin liquids. Correct Answer: Vitamin D

Rationale: Dysphagia means difficulty with Rationale: The elderly have greater need for
swallowing. Liquids that are thickened are easier to vitamin D due to decreased absorption, decreased
swallow and less likely to cause the client to choke dietary intake, and lack of exposure to sunlight.
or gag. The frequency or temperature of foods does Calcium, protein, and vitamin K are more readily
not affect the ability to swallow. obtained through dietary means.
Nursing Process: Implementation Nursing Process: Evaluation
Cognitive Level: Application Cognitive Level: Comprehension
NCLEX-RN Test Plan: Physiological Integrity NCLEX-RN Test Plan: Health Promotion and
________________________________________ Maintenance
10. ________________________________________
A primary goal of dietary intervention for the elderly 12.
is to: The loss of lean muscle mass that occurs with aging
Your Answer: Improve digestion and can be diminished or reversed by:
metabolism. Your Answer: Small doses of anabolic
Correct Answer: Maintain quality of life. hormones.
Correct Answer: Strength-training exercises.
needed.
Rationale: Resistance exercises, such as
lifting weights, have been shown to increase muscle Rationale: The use of adaptive equipment
mass in the elderly. Anaerobic exercises, high- allows residents to be as independent as possible at
protein diets, and anabolic steroids are not effective mealtime. It is important to allow clients to do as
in reducing loss of muscle mass or increasing much as possible for themselves.
muscle mass. Nursing Process: Implementation
Cognitive Level: Application
Nursing Process: Evaluation NCLEX-RN Test Plan: Safe, Effective Care
Cognitive Level: Application Environment
NCLEX-RN Test Plan: Physiological Integrity ________________________________________
________________________________________ 15.
13. A major predictor of morbidity and mortality in the
The absorption of iron in the elderly can be elderly is:
diminished in the presence of: Your Answer: Polypharmacy.
Your Answer: Vitamin C. Correct Answer: Unintentional weight loss.
Correct Answer: Antacids.
When an elderly male client states he has
Rationale: Anything that interferes with the pain, the nurse recognizes that:
production of hydrochloric acid interferes with the Your Answer: The client probably has
absorption of iron, and should be avoided when significant pain, because many elderly understate
trying to promote iron absorption. Diuretics, vitamin and under-report pain.
C, and milk do not affect iron absorption.
Nursing Process: Implementation Rationale: Many elderly fear being labeled as
Cognitive Level: Knowledge complainers, so they frequently fail to mention
NCLEX-RN Test Plan: Physiological Integrity having pain, or diminish its significance to them.
________________________________________ The elderly are no more prone to exaggerate pain
14. than is any other age group. The elderly do not lose
When serving a meal to elderly residents of a pain receptors, and the most frequent reason for
nursing home, the nurse should: healthcare visits is to manage chronic health
Your Answer: Open cartons and cut food into conditions.
bite-sized portions. Nursing Process: Assessment
Correct Answer: Provide adaptive equipment as Cognitive Level: Application
NCLEX-RN Test Plan: Physiological Integrity
________________________________________ Rationale: This federal law governs anyone’s
2. ability to give informed consent after being fully
When the family of an elderly resident at the informed about the options, benefits, or risks of any
nursing home wants to file a complaint about the given treatment. The other options might be
improper use of restraints, the nurse should suggest desirable; they are not part of the Patient Self-
that they: Determination Act of 1990.
Your Answer: Contact the attorney retained by Nursing Process: Planning
the nursing home administrator. Cognitive Level: Knowledge
Correct Answer: Call the ombudsman assigned to NCLEX-RN Test Plan: Safe, Effective Care
the nursing home. Environment
________________________________________
Rationale: The improper use of restraints is a 4.
valid reason for contacting the nursing home Which of the following will the nurse want to include
ombudsman, whose name and phone number when planning a functional assessment of an elderly
should be readily available to residents and families. client?
The other options do not identify the appropriate Your Answer: A social worker
individual to handle the family’s request to file a
complaint. Rationale: A functional assessment is
Nursing Process: Evaluation conducted by an interdisciplinary team, and may be
Cognitive Level: Knowledge performed while a client is an inpatient at a
NCLEX-RN Test Plan: Safe, Effective Care hospital, but also may be conducted in an office
Environment setting. All of the other things might be nice to have
________________________________________ when the assessment is conducted, but the nurse
3. does not need to have them.
The nurse who understands the Patient Self- Nursing Process: Planning
Determination Act of 1990 understands that: Cognitive Level: Application
Your Answer: Medicare coverage is extended NCLEX-RN Test Plan: Health Promotion and
for those elderly who suffer from catastrophic Maintenance
conditions. ________________________________________
Correct Answer: Elderly clients who are 5.
competent have the right to make informed When a nursing home uses the Minimum Data Set
decisions regarding healthcare. (MDS), the nurse is assured that:
Your Answer: Regular dental care is being Which of the following nursing actions will help
provided. maintain safety for the elderly?
Correct Answer: The nursing home is meeting Your Answer: Encourage independence.
federal regulations. Correct Answer: Orient to new surroundings.

Rationale: The MDS provision of the Omnibus Rationale: Safety is a priority need for the
Budget Reconciliation Act of 1987 requires elderly, especially when in new surroundings, such
assessment of all residents using the criteria of the as a hospital or long-term care facility; they can
MDS. The MDS does not address the other options. become disoriented when they cannot easily locate
Nursing Process: Evaluation familiar places like the bathroom. The nursing
Cognitive Level: Analysis actions in the other actions are desirable but are not
NCLEX-RN Test Plan: Safe, Effective Care primarily for client safety.
Environment Nursing Process: Implementation
________________________________________ Cognitive Level: Application
6. NCLEX-RN Test Plan: Safe, Effective Care
The primary purpose for documenting an adverse Environment
event, such as a fall, is to: ________________________________________
Your Answer: Record unusual occurrences. 8.
Correct Answer: Improve the quality of care. The nurse observes the following actions and
recognizes that a breech of confidentiality is evident
Rationale: Adverse events are always when a colleague:
opportunities for quality improvement, and Your Answer: Uses the phone to confer with a
documentation should lead to system analysis and physician’s nurse about a client’s change of
improvement. Careful records need to be kept to condition.
document follow-up with respect to staff Correct Answer: Throws unofficial lab results into
development and client outcomes; however, the the regular trash.
ultimate goal is to improve the quality of care.
Nursing Process: Evaluation Rationale: Client information may be shared
Cognitive Level: Analysis with those who need to know it. When information
NCLEX-RN Test Plan: Safe, Effective Care with a client’s name is thrown in the regular trash, it
Environment might be found and read by anyone.
________________________________________ Nursing Process: Evaluation
7. Cognitive Level: Application
NCLEX-RN Test Plan: Safe, Effective Care relationships. The other choices presented are dealt
Environment with in a functional assessment.
________________________________________ Nursing Process: Assessment
9. Cognitive Level: Knowledge
When an elderly person agrees with a physician that NCLEX-RN Test Plan: Psychosocial Integrity
a do not resuscitate (DNR) order is appropriate, that
person is said to be: Primary prevention of elder mistreatment is a
Your Answer: In need of a living will. healthcare goal. The nurse recognizes that one of
Correct Answer: Competent. the risk factors often present in elder mistreatment
situations is:
Rationale: When a physician explains to a Your Answer: Caregiver independence.
client that additional care is futile, the client who is Correct Answer: Elder dependency.
competent might not agree. Based on the wishes of
the competent client, an order may be written. Rationale: One of the primary characteristics
Giving assent suggests that the client cannot give in elders who have been mistreated is dependency
fully informed consent. The competent client has full on others for performance of activities of daily
decisional capacity, and does not need a living will living. Dependence can lead to caregiver strain and
unless he so chooses. then mistreatment. Home care is not a risk factor
Nursing Process: Implementation for mistreatment. Caregiver independence is not
Cognitive Level: Application related to mistreatment of the elderly. Elders who
NCLEX-RN Test Plan: Safe, Effective Care have minimal family support can feel abandoned.
Environment Nursing Process: Analysis
________________________________________ Cognitive Level: Comprehension
10. NCLEX-RN Test Plan: Psychosocial Integrity
When obtaining a social history from an elderly ________________________________________
client, the nurse will ask about: 2.
Your Answer: Problems with memory, A home care nurse found an elderly client with
judgment, and thought. multiple bruises on both arms and the trunk. After
Correct Answer: Living arrangements and family the client denied the spouse was responsible, the
dynamics. rationale the nurse used for reporting the suspicion
of abuse was:
Rationale: A social history has to do with Your Answer: The client was too afraid to
social support systems and interpersonal report the spouse.
Correct Answer: Reporting suspected abuse is an Nursing Process: Implementation
ethical responsibility of the nurse. Cognitive Level: Analysis
NCLEX-RN Test Plan: Safe, Effective Care
Rationale: The nurse has an ethical and legal Environment
responsibility to report suspected abuse to ________________________________________
designated authorities for investigation. Because 4.
the nurse has this responsibility, legal liability is not Which of the following situations could be the
an issue, nor are the denials by the client grounds source of an ethical dilemma for the nurse with
for not reporting the suspicion of abuse. respect to elder mistreatment?
Nursing Process: Analysis Your Answer: The family sues the nurse for
Cognitive Level: Application reporting abuse.
NCLEX-RN Test Plan: Safe, Effective Care Correct Answer: The elderly person decides to
Environment return to the abusive setting.
________________________________________
3. Rationale: The nurse recognizes an ethical
When a nurse determines with a high degree of dilemma is posed when a competent adult chooses
confidence that an elderly homebound client has to return to the setting in which abuse occurred,
been mistreated, an appropriate intervention would because there is a high likelihood of abuse being
be: repeated. The elderly person who is not competent
Your Answer: Consulting with the physician will not be allowed to make decisions about the
about admission to the hospital for a thorough living situation. Injury is not a source of an ethical
assessment of the client. dilemma, nor is legal action taken against a nurse.
Nursing Process: Assessment
Rationale: It is appropriate to admit a client to Cognitive Level: Application
the hospital for a thorough assessment in a safe NCLEX-RN Test Plan: Safe, Effective Care
environment, and it serves as an immediate way to Environment
remove the client from the situation. Social Services ________________________________________
will conduct an investigation, but the nurse has a 5.
responsibility to formulate a plan of care for the An elderly person reports all of the following
client. The caregiver might not be responsible for experiences to the nurse. Which of the reported
the abuse, so it would not be appropriate to suggest experiences is the most likely indicator of
a new caregiver. Police surveillance is not done for exploitation of the elderly person?
suspected mistreatment. Your Answer: The client is being threatened
with admission to a nursing home. ________________________________________
Correct Answer: There isn’t any money available 7.
to make a weekly donation at church. What data would help support a nursing diagnosis of
elder mistreatment in the domestic setting?
Rationale: When the elderly are exploited, Your Answer: Client has a flat affect.
others take advantage of them, almost always with Correct Answer: Caregiver will bathe the client
respect to money matters. The exploiters might not only once a week.
be family members. Failure of family to visit and
forced attendance of family events are not Rationale: For the elderly, some hygiene
exploitation. Threats can be attempts at coercion measures might not be needed or desirable in the
but are not exploitation. same frequency as for younger adults. However,
Nursing Process: Analysis bathing only once weekly is not likely to be
Cognitive Level: Analysis sufficient. Unhappiness with living arrangements
NCLEX-RN Test Plan: Psychosocial Integrity and a flat affect are not evidence of mistreatment.
________________________________________ Recognition of challenges with care giving does not
6. mean mistreatment has occurred.
The nurse suspects that a home health aide who Nursing Process: Assessment
comes into the home every day to provide basic Cognitive Level: Analysis
hygiene care is abusing an elderly client. What is NCLEX-RN Test Plan: Safe, Effective Care
the best way to gather more information about the Environment
situation to confirm suspicions of abuse? ________________________________________
Your Answer: Interview the client in private. 8.
Mistreatment that occurs in nursing homes and is
Rationale: The best method of getting directed toward the elderly often is due to:
information is to interview the client in a private Your Answer: Staff’s lack of interest in the
setting. Depending on the responses of the client, elderly as individuals.
the nurse can develop a plan for how to proceed. Correct Answer: Low wages and heavy workloads
The nurse then can consult the family or involve a of the nursing assistants.
social services agency.
Nursing Process: Assessment Rationale: Individuals who have few skills and
Cognitive Level: Application inadequate education in the complex care needs of
NCLEX-RN Test Plan: Health Promotion and the elderly, along with accompanying low wages
Maintenance and heavy workloads, might mistreat the elderly out
of simple frustration. on the arm. Nursing Process: Assessment
Nursing Process: Evaluation Cognitive Level: Analysis
Cognitive Level: Analysis NCLEX-RN Test Plan: Psychosocial Integrity
NCLEX-RN Test Plan: Safe, Effective Care
Environment An elderly client whose middle-age daughter
________________________________________ recently died of breast cancer now complains of
9. mild abdominal pain, five-pound weight loss,
A nurse who plans to do research on caregiver insomnia, and fatigue. When no physiological cause
strain in spouses of homebound elderly with can be found, the nurse suspects these are
cognitive impairment due to dementia is most likely symptoms of:
to choose which theory of elder mistreatment to Your Answer: Hypochondria.
guide the study? Correct Answer: Normal grieving.
Your Answer: Situational theory
Rationale: Physiological symptoms are a
Rationale: Situational theory examines the normal part of grieving, particularly in the early
burdens of caregiving and the ability of the phases. In the absence of pathology, these are signs
caregiver to meet the needs and demands of the of normal grieving. Denial, spiritual distress, and
elderly individual. Mistreatment can result when the hypochondria usually do not manifest themselves
situation becomes overwhelming for the caregiver. with physical symptoms.
The other theories do not relate to caregiver strain. Nursing Process: Evaluation
Nursing Process: Planning Cognitive Level: Analysis
Cognitive Level: Application NCLEX-RN Test Plan: Psychosocial Integrity
NCLEX-RN Test Plan: Health Promotion and ________________________________________
Maintenance 2.
________________________________________ A nurse who works effectively with elderly clients
10. who are dying and their families recognizes that:
Which of the following observations is indicative of Your Answer: At least some pain accompanies
physical mistreatment? most deaths.
Your Answer: Downcast eyes Correct Answer: The nurse must be comfortable
Correct Answer: Dislocated shoulder with her own concerns and feelings about death.

Rationale: This is the only listed physical sign. Rationale: Nurses and other healthcare
A dislocated shoulder can occur with a violent jerk providers who are comfortable with their own
understanding of death can be effective when Your Answer: Assist the bereaved individual to
working with the dying, rather than bringing their achieve a healthy adjustment to the loss.
conflicted feelings to a stressful situation. Unless
the nurse has used some critical thinking to look at Rationale: The goal of grieving is to adjust to
and investigate her feelings and understandings, the loss in a time and manner that is culturally
they might conflict with those of the client. Many acceptable to the individual who experienced the
people are indeed afraid of dying, especially if pain loss. There is no timetable that must be met, no
might occur. However, pain is not inevitably defined way to achieve the goal. An intervention to
associated with death. Hospice might not be progress toward the goal is verbalization about the
appropriate or possible for many individuals. loved one. Teaching is a nursing goal and is not
Nursing Process: Analysis client-centered.
Cognitive Level: Comprehension Nursing Process: Planning
NCLEX-RN Test Plan: Psychosocial Integrity Cognitive Level: Comprehension
________________________________________ NCLEX-RN Test Plan: Psychosocial Integrity
3. ________________________________________
It is important for a nurse to understand the 5.
grieving process because: An elderly person in end-stage renal disease is
Your Answer: It assists the nurse to admitted to a nursing home for palliative care.
understand the dynamics of grieving. Nursing interventions will be:
Your Answer: Ambulation as desired.
Rationale: Grief is unique to each individual Correct Answer: Pain relief.
and family and culture. By understanding the
grieving process, the nurse will recognize the social, Rationale: The purpose of palliative care is
cultural, and physiological dynamics present during pain relief when no additional treatment is desired
grieving. There is no optimal order in which or available. Comfort measures direct the primary
individuals must move through the grieving process. nursing intervention, and can include medications or
Nursing Process: Evaluation complementary therapies, such as massage.
Cognitive Level: Comprehension Individuals receiving palliative care are not
NCLEX-RN Test Plan: Psychosocial Integrity candidates for CPR. Urine output is not expected in
________________________________________ end-stage renal disease.
4. Nursing Process: Planning
The goal of nursing interventions for a bereaved Cognitive Level: Application
elderly person is to: NCLEX-RN Test Plan: Physiological Integrity
________________________________________ client may be on other medications concurrently.
6. Nursing Process: Evaluation
A nurse is uncomfortable discussing spiritual Cognitive Level: Comprehension
concerns with a dying client. The most helpful action NCLEX-RN Test Plan: Physiological Integrity
for the client would be for the nurse to plan to: ________________________________________
Your Answer: Request a member of the 8.
pastoral care staff visit the client. A priority nursing intervention for an elderly person
who is dying and experiencing anxiety is to:
Rationale: Pastoral care is available at most Your Answer: Allow the client time alone to
sites, and if a nurse is uncomfortable with spiritual conduct a life review.
issues, the pastoral care services might be better Correct Answer: Assist the individual to identify
able to meet the spiritual needs of the client. A fears.
nurse would plan to get education, rather than
counseling, on how to meet the needs of dying Rationale: People who are dying have many
clients. Discomfort with a situation is not a reason fears, including pain, being left alone, or being
for asking to be removed from caring for that client. forgotten. The nurse who can help the client identify
Nursing Process: Planning those fears then can work with the client to explain
Cognitive Level: Application what can be done to alleviate the concerns. Some
NCLEX-RN Test Plan: Psychosocial Integrity time alone might be appropriate, but many times, it
________________________________________ increases anxiety. Contacting family might be
7. appropriate, but not until after the nurse has
When opioids are prescribed for pain at the end of determined with the client that it will be helpful.
life, the nurse should understand that: Merely explaining that anxiety is common, without
Your Answer: Opioids most likely will be a PRN further intervention, is not the best action the nurse
order. can take.
Correct Answer: Side effects still must be treated. Nursing Process: Implementation
Cognitive Level: Application
Rationale: Opioids are the gold standard of NCLEX-RN Test Plan: Psychosocial Integrity
pain relief; however, side effects like constipation ________________________________________
are common, and must be treated to provide 9.
comfort to the client. A client can be on opioids for a Which of the following statements, if made by a
long time before death; they are most effective dying client, would indicate that spiritual needs
when given on an around-the-clock schedule. A most likely are being met? The individual states
that: do.
Your Answer: “There have been many positive Nursing Process: Implementation
things about my life, and I have hope.” Cognitive Level: Application
NCLEX-RN Test Plan: Psychosocial Integrity
Rationale: When an individual’s spiritual
needs have been met, the person can be expected
to express feelings of satisfaction with one’s life and
develop a new understanding of hope. Many people When administering medication to an elderly
who do not subscribe to particular spiritual beliefs hospitalized client, the nurse should be aware that
might not fear pain, and many have an appreciation there is an increased possibility of drug toxicity due
for family. Some religious groups do not believe in to which of the following age-related changes?
an afterlife. Your Answer: Decreased filtration and
Nursing Process: Evaluation excretion of drug metabolites through the renal
Cognitive Level: Analysis system
NCLEX-RN Test Plan: Psychosocial Integrity
________________________________________ Rationale: Glomerular filtration and creatinine
10. clearance begin to decline in middle age, and can
When an elderly client expresses a wish to forgo be significantly lower in most elderly, posing a risk
additional treatment for cancer and to die, a priority of greater drug toxicity. There is no increase in
action of the nurse would be: protein-binding sites or circulating body fluids in the
Your Answer: Explore the client’s elderly. Production of enzymes also is not decreased
understanding of the consequences of such a with aging.
decision. Nursing Process: Planning
Cognitive Level: Application
Rationale: When respecting a client’s NCLEX-RN Test Plan: Physiological Integrity
autonomy, the nurse recognizes that the client has ________________________________________
a right to make such a decision. It is important to 2.
make sure the client understands what such a Which of the following indicates that an elderly
decision means about the outcome of the disease client has been affected by polypharmacy?
process. At some point, the physician and family Your Answer: Medications are used to
should be notified, but only after discussion with the counteract side effects of currently prescribed
client affirms the decision. The role of the nurse is to medications.
support a client, not suggest what the client should
Rationale: Polypharmacy means that multiple Maintenance
medications have been prescribed. When ________________________________________
medications are prescribed to counteract side 4.
effects of other medications, it is an indication that What is frequently a contributing factor to the
a total review of medications should occur to elderly using multiple medications?
determine the optimum medications and dosages Your Answer: Increased challenges in
with the fewest side effects. Lack of improvement in compliance with medication regimens
a condition might mean another evaluation is Correct Answer: Healthcare providers’ failure to
warranted. Healthcare providers need to determine coordinate medication regimens
the possible benefits and risks in prescribing or not
prescribing any given medication. Rationale: The elderly client might be under
Nursing Process: Evaluation the care of multiple healthcare providers who are
Cognitive Level: Analysis unaware of the medications the other providers
NCLEX-RN Test Plan: Health Promotion and have prescribed. The elderly should be encouraged
Maintenance to maintain a list of medications, dosages,
________________________________________ frequency, and prescriber information to share with
3. all healthcare providers. The greater the number of
In assessing use of over-the-counter (OTC) medications the client is taking, the higher the
medications, which of the following questions will likelihood of side effects and decreased compliance
the nurse want to ask the elderly client? with medication regimens.
Your Answer: “Who pays for the medication?” Nursing Process: Evaluation
Correct Answer: “How does this medication help Cognitive Level: Analysis
you?” NCLEX-RN Test Plan: Health Promotion and
Maintenance
Rationale: It is important to assess how the ________________________________________
medication helps the client or how the client feels it 5.
is helpful. Payment or brand selections are up to the Anticholinergic effects of antipsychotic medications
client, and might be related to other considerations. in the elderly are associated with:
Other treatment strategies do not relate to OTC Your Answer: Cognitive impairment.
medications. Correct Answer: Difficulty with voiding.
Nursing Process: Assessment
Cognitive Level: Application Rationale: Anticholinergic effects commonly
NCLEX-RN Test Plan: Health Promotion and are found in prescribed medications, and frequent
side effects (urinary retention, dry mouth, blurred increased only if necessary.
vision, decreased peristalsis, etc.) occur in the Nursing Process: Planning
elderly, so the nurse needs to assess for those side Cognitive Level: Comprehension
effects with elderly clients. NCLEX-RN Test Plan: Health Promotion and
Nursing Process: Assessment Maintenance
Cognitive Level: Knowledge ________________________________________
NCLEX-RN Test Plan: Physiological Integrity 8.
________________________________________ The ultimate goal of prescribing antipsychotic
6. medications for the elderly is:
Gastritis in the elderly frequently has been Your Answer: To relieve stress on family
associated with: members who are caregivers.
Your Answer: Beta blockers. Correct Answer: To improve the quality of life.
Correct Answer: Low doses of aspirin.
Rationale: Antipsychotic medications should
Rationale: Aspirin is highly irritating to the be prescribed with great caution in the elderly and
stomach, and can lead to heartburn or gastritis, should be viewed only as short-term therapy to
even with low-dose or enteric coating, if it is not improve quality of life.
taken properly. Other types of medications listed Nursing Process: Implementation
are not frequently associated with gastritis. Cognitive Level: Analysis
Nursing Process: Implementation NCLEX-RN Test Plan: Psychosocial Integrity
Cognitive Level: Application ________________________________________
NCLEX-RN Test Plan: Physiological Integrity 9.
________________________________________ An elevated level of a highly protein-bound
7. medication might be a result of which
General guidelines regarding prescribing pharmacokinetic alteration in the elderly?
medications for the elderly clients are: Your Answer: Increased CPK
Your Answer: Start low and go slow. Correct Answer: Decreased serum albumin

Rationale: This rule of thumb is followed to Rationale: Decreased serum albumin results
prevent toxic effects and adverse drug reactions. in greater amounts of the unbound proportion of
The usual practice is to start with the lowest dose protein-bound medications. This can result in toxic
possible and evaluate regularly to see if symptoms levels, because the unbound portion is available to
subside or the condition improves; the dose is produce effects. The other substances do not affect
protein-bound medications. with a written copy to take home.
Nursing Process: Planning
Cognitive Level: Knowledge Rationale: Oral instruction allows the nurse to
NCLEX-RN Test Plan: Health Promotion and answer questions and to verify client understanding
Maintenance of instructions. Written information is for the client
________________________________________ or family to review if they have additional questions
10. or need more information. Clients might seek other
The area in which the nurse may have the greatest sources of information, but it is the responsibility of
effect on medication use in the elderly is: the nurse to provide information that is
Your Answer: Screening for side effects. understandable to the client.
Correct Answer: Client education. Nursing Process: Implementation
Cognitive Level: Application
Rationale: Teaching clients and families about NCLEX-RN Test Plan: Health Promotion and
safe practices and information about side effects Maintenance
will ensure medications that are effective are used ________________________________________
correctly and have the fewest effects. The nurse 12.
may encourage alternative therapies if they do not An elderly client asks why he seems to respond
affect medication compliance or increase the risk of differently to some medications than he did in the
side effects. Monitoring clients is important, but past. The nurse should keep in mind that:
teaching is likely to have the greatest impact on Your Answer: In the elderly, an adverse drug
clients. reaction can occur even after the medication has
Nursing Process: Implementation been discontinued.
Cognitive Level: Analysis
NCLEX-RN Test Plan: Health Promotion and Rationale: Due to the pharmacokinetics and
Maintenance pharmacodynamics in the elderly, it is possible to
________________________________________ have adverse reactions during or following
11. discontinuation of medication therapy. Glomerular
When developing a teaching plan for an elderly filtration is decreased in the elderly, so there might
client newly diagnosed with hypertension, the nurse be more side effects.
realizes that the most effective teaching strategy is: Nursing Process: Evaluation
Your Answer: Sharing the Web address of the Cognitive Level: Analysis
American Heart Association. NCLEX-RN Test Plan: Health Promotion and
Correct Answer: Providing oral instructions along Maintenance
________________________________________ constriction should be symmetrical. There should be
13. a red reflex, and the eyelids and conjunctivae
If an elderly client tells the nurse about medications should not be edematous or inflamed.
from an internet site with NAPB VIPPS (National Integrated Process: Nursing Process; Assessment
Association of Boards of Pharmacy, Verified Internet Cognitive Level: Analysis
Pharmacy Practice Site), the nurse can be fairly NCLEX-RN Test Plan: Physiological Integrity;
certain that: Reduction of Risk
Your Answer: The client has financial need. ________________________________________
Correct Answer: The site meets state and federal 2.
requirements. Which of the following physiologic changes would be
expected in a patient with presbyopia?
Rationale: Increasing use of the internet by Your Answer: Loss of lens elasticity
the elderly and others to purchase medications has
led to federal and state regulations that assure Rationale: The lens loses elasticity or
safety for the consumer. Medications purchased pliability, decreasing its ability to change shape in
from the internet are not necessarily from foreign order to accommodate for near vision. Response 4
countries, and do not necessarily mean the client is is indicative of cataracts, 3 is myopia, and 1 is due
experiencing financial distress. to corneal changes.
Nursing Process: Evaluation Integrated Process: Nursing Process; Assessment
Cognitive Level: Analysis Cognitive Level: Analysis
NCLEX-RN Test Plan: Health Promotion and NCLEX-RN Test Plan: Physiological Integrity;
Maintenance Reduction of Risk Potential
________________________________________
You are the nurse assigned to perform an eye 3.
assessment on an 80-year-old client. Which of the A patient has a question about a recent eye exam.
following findings during the assessment is Which of the following statements would be an
considered normal? accurate response to inquiry?
Your Answer: Absence of the red reflex Your Answer: "The lens is normally
Correct Answer: Equal pupillary constriction in transparent."
response to light
Rationale: The lens is made of layers of
Rationale: Pupils are small in old age, and the epithelial cells that are closely packed and
pupillary light reflex can be slowed, but pupillary organized so that the lens is clear and transparent.
Hypermetropia is a shortened eyeball, and myopia to decrease intraocular pressure. Which of the
is an elongated eyeball. Refraction refers to bending following medication should be questioned by the
the light rays so they are focused at a certain point nurse?
on the retina. Your Answer: Artificial tears
Integrated Process: Teaching Learning
Cognitive Level: Application Rationale: Timoptic, levobunolol, and
NCLEX-RN Test Plan: Health Promotion and pilocarpine are drugs usually prescribed for patients
Maintenance with glaucoma and increased intraocular pressure
________________________________________ (IOP). Artificial tears are usually prescribed for
4. persons with "dry eyes."
A client comes to the office with a tentative Integrated Process: Nursing Process; Evaluation
diagnosis of glaucoma, and is to have testing to Cognitive Level: Evaluation
determine whether the anterior angle of the eye is NCLEX-RN Test Plan: Physiological Integrity;
open or closed. The patient will require education Pharmacological and Parenteral Therapies
for which of the following examinations? ________________________________________
Your Answer: Retinography 6.
Correct Answer: Slit-lamp inspection A client with history of glaucoma was diagnosed by
the community nurse as experiencing Visual
Rationale: The slit-lamp inspection of the iris Sensory/Perceptual Alterations R/T increased
is done to assess whether the anterior angle of the intraocular pressure. The plan of care should focus
eye is open or closed. This is part of a complete on:
dilated examination to inspect the optic nerve and Your Answer: Encouraging compliance with
retina. Tonometry measures only the pressure drug therapy to prevent loss of vision.
within the eye. Response 2 would be used to take
multiple x-rays of the eye tissue, and retinography Rationale: Management of glaucoma involves
records the electrical activity of the eye. lowering the intraocular pressure to cease drainage
Integrated Process: Teaching Learning to the optic nerve and further vision loss. Therapy
Cognitive Level: Application involves medications to decrease intraocular
NCLEX-RN Test Plan: Health Promotion and pressure. It is critical that the patient understand
Maintenance the importance of compliance with the medication
________________________________________ regimen. Glaucoma needs to be controlled to
5. prevent damage that cannot be reversed. Pain is
A patient with glaucoma has medication prescribed considered an emergency, so if it was identified, the
patient would need immediate intervention. Loss of Cognitive Level: Analysis
peripheral vision is a concern, but the priority is to NCLEX-RN Test Plan: Physiological Integrity;
encourage the patient to be compliant with Adaptation
medications to prevent further damage to the ________________________________________
vision. 8.
Integrated Process: Nursing Process; You are the nurse assigned to care for an elderly
Implementation client. While assessing the ears of the patient, you
Cognitive Level: Synthesis observed dry, hard cerumen developing in the ear
NCLEX-RN Test Plan: Health Promotion and canal. Which of the following actions should you
Maintenance take?
________________________________________ Your Answer: Irrigate the ear canal with a
7. mixture of hydrogen peroxide and normal saline.
A 76-year-old client tells the nurse that she notices Correct Answer: Document finding and report
that she is having trouble hearing, especially in your concern to the physician.
large groups. She continues, saying she cannot
always tell where the sound is coming from, and the Rationale: Cerumen or earwax produced by
words often sound "mixed up." The nurse should the ceruminous glands is a normal finding in the ear
suspect which of the following as the cause for this canal. However, in the older adult, cerumen tends to
change? be dryer and harder, and accumulates in the ear
Your Answer: Nerve degeneration in the inner canal due to decreased activity of the apocrine
ear glands. Hearing will become impaired if the
cerumen accumulates to impact the canal. Irrigation
Rationale: Hearing loss is common in older should be tried after other attempts fail. Therefore,
adults. Changes in the inner ear involve atrophy of documentation and reporting to the primary care
the organ of corti and cochlear, and degeneration of provider will help avoid unnecessary alteration in
the stria vascularis. This is a gradual hearing loss hearing.
that affects the auditory nerve, which makes words Integrated Process: Nursing Process; Analysis
sound garbled, and the ability to localize sound is Cognitive Level: Analysis
also impaired. The result is that communication NCLEX-RN Test Plan: Physiological Integrity; Basic
dysfunction is accentuated when background noise Care and Comfort
is present. Responses 1, 2, and 4 would not affect ________________________________________
the sensorineural component of hearing. 9.
Integrated Process: Nursing Process; Analysis During a hearing assessment, the nurse finds that
sound lateralizes to the client's left ear with the Rationale: Presbycusis is hearing loss
Weber and Rinne tests. What should the nurse associated with aging, and affects approximately
conclude from this finding? The patient has: 75% of people over the age 60. Loss of hair cells in
Your Answer: A conductive hearing loss in the the cochlea (sensory loss) and degradation of
right ear. neurons (neural loss) result in this form of hearing
Correct Answer: Either a sensorineural or loss. Presbycusis occurs gradually, and usually is
conductive hearing loss. bilateral. Answer 3 describes a conductive hearing
loss.
Rationale: When caring for the older person, a Integrated Process: Nursing Process; Analysis
thorough history and physical of the person's Cognitive Level: Application
hearing is important to help determine the cause of NCLEX-RN Test Plan: Physiological Integrity;
hearing loss, which can be conductive, due to Reduction of Risk Potential
external aspect of the ear, or sensorineural, from an ________________________________________
inner ear problem. 11.
Both tests determine the degree of hearing loss, if An 84-year-old client is admitted to the hospital for
any. treatment of diabetes mellitus.
Integrated Process: Nursing Process; Assessment On initial interview, the patient reports having
Cognitive Level: Analysis difficulty hearing, and informs the nurse that she
NCLEX-RN Test Plan: Physiological Integrity: has too much ear wax. Because of the patient's age,
Reduction of Risk Potential which of the following questions is most important
________________________________________ for the nurse to ask the client in relation to her
10. hearing problem?
Which of the following is true of presbycusis? (Select Your Answer: "What type of diabetes mellitus
all that apply.) do you have?"
Your Answer: Dysfunction with the cochlea Correct Answer: "Have you ever experienced
and auditory nerve impacted cerumen?"
Correct Answers: Dysfunction with the cochlea
and auditory nerve Rationale: Because the hairs in the ear
become coarser during the aging process, retention
Ménière’s disease and tumors of wax is more of a cause of hearing loss in older
clients. Cerumen, if dry and hard, can impact the
Infections of the inner ear ear canal and interfere with hearing.
Integrated Process: Nursing Process; Assessment
Cognitive Level: Analysis pain upon swallowing. Xerostomia refers to dry
NCLEX-RN Test Plan: Health Promotion and mouth that occurs with salivary gland dysfunction.
Maintenance Arthralgia refers to pain in a joint.
________________________________________ Integrated Process: Communication and
12. Documentation
Which of the following drugs are not associated with Cognitive Level: Analysis
impairment? Select all that apply. NCLEX-RN Test Plan: Safe, Effective Care
Your Answer: Gentamicin (Garamycin) Environment
Correct Answers: Gentamicin (Garamycin) ________________________________________
14.
Cis-platinum (Platinol) A client has noticed a decrease in taste sensation.
Which of the following cranial nerves are most likely
Furosemide (Lasix) involved?
Your Answer: CN IX and CN VII
Rationale: Digoxin (Lanoxin) causes vision
changes, but does not affect hearing. Rationale: Cranial nerves VII and IX supply
Aminoglycosides such as gentamicin, sensation to the tongue.
antineoplastics such as cis-platinum, diuretics such Integrated Process: Nursing Process; Analysis
as furosemide, antispasmodics such as baclofen, Cognitive Level: Analysis
and propranolol cause changes in hearing ability. NCLEX-RN Test Plan: Physiological Integrity;
Integrated Process: Nursing Process; Analysis Reduction of Risk Potential
Cognitive Level: Analysis ________________________________________
NCLEX-RN Test Plan: Physiological Integrity; 15.
Pharmacological and Parental Therapies A 78-year-old client is admitted to the Emergency
________________________________________ Department (ED) via emergency medical service
13. (EMS) with complaints of severe diarrhea with
A patient has a normal sensory change that results resultant weakness and signs of dehydration.
in diminished sense of taste. How would this be Discussion with the significant other reveals that the
documented? patient continually eats spoiled foods. Which of the
Your Answer: Hypogeusia following might be most directly related to this
patient's behavior?
Rationale: A diminished sense of taste is Your Answer: Damage to cranial nerve III
referred to as hypogeusia. Odynophagia refers to Correct Answer: Damage to cranial nerve I
Correct Answer: The dermis contains nerves and
Rationale: Although hyposmia (impaired sensory receptors.
sense of smell) might be due to age-related
changes, it might also be the result of olfactory Rationale: The dermis, the second layer of the
nerve damage (cranial nerve I), since this nerve is skin, is made up of connective tissue, collagen, and
the sole innervation for smell. Cranial nerve II is has resilient elastic tissue that allows the skin to
responsible for constriction of the pupils, and IV the stretch. It is the thickest skin layer and also contains
superior oblique muscle. Cranial II would affect fibroblasts, mast cells, and lymphocytes.
vision, but an intact CN I would allow the individual Integrated Process: Nursing Process; Planning
to identify the odor of spoiled food accurately. Cognitive Level: Knowledge
Integrated Process: Nursing Process; Analysis NCLEX-RN Test Plan: Health Maintenance and
Cognitive Level: Analysis Promotion
NCLEX-RN Test Plan: Physiological Integrity; ________________________________________
Physiological Adaptation 2.
________________________________________ During the aging process, the hair can look gray or
16. white and begin to feel thin and fine. This is due to
In evaluating the functioning of a client's anterior decreased number of functioning:
spinothalamic tract, the nurse is testing for: Your Answer: Lymphocytes.
Your Answer: Sense of position. Correct Answer: Melanocytes.
Correct Answer: Light touch sensation.
Rationale: In the aging hair matrix, the
Rationale: It is the spinothalamic tract that number of functioning melanocytes decreases, so
transmits light (crude) touch sensation. the hair looks gray or white and feels thin and fine.
Integrated Process: Evaluation The phagocytes, lymphocytes, and eosinophils are
Cognitive Level: Knowledge cells important to the inflammatory response and
NCLEX-RN Test Plan: Health Promotion and the immune system.
Maintenance Integrated Process: Nursing Process: Analysis
Cognitive Level: Knowledge
The nurse is teaching a group of elderly women NCLEX-RN Test Plan: Physiological Integrity;
about skin care. Which of the following would be Reduction of Risk Potential
accurate to include in the content? ________________________________________
Your Answer: The dermis is completely 3.
replaced every four weeks. An 84-year-old patient's skin is assessed to be
wrinkled, thin, and dry. These findings should be Cognitive Level: Application
interpreted as related to: NCLEX-RN Test Plan: Physiological Integrity;
Your Answer: Increased vascularity of the skin. Reduction of Risk Potential
Correct Answer: Loss of elastin and a decrease in ________________________________________
subcutaneous fat. 5.
Which of the following is accurate about skin
Rationale: Several factors cause wrinkled, cancer? Skin cancers that result primarily from sun
thin, dry skin. These include gradual loss of elastin; exposure are:
decreased subcutaneous fat; decreased number of Your Answer: Hodgkin's disease.
epithelial cells providing a barrier, causing Correct Answer: Basal cell carcinomas.
insensible loss of body fluids from the deeper layers
of the skin; a lifetime of environmental trauma to Rationale: Skin cancers that result primarily
the skin; increasing sedentary lifestyle; and from sun exposure are basal cell carcinoma (BCC),
decrease in mobility. squamous cell carcinoma (SCC), and malignant
Integrated Process: Nursing Process; Analysis melanoma (MM). The risk for squamous cell
Cognitive Level: Application carcinoma is strongly associated with chronic sun
NCLEX-RN Test Plan: Physiological Integrity; exposure but not with intermittent exposure.
Reduction of Risk Potential Hodgkin's disease is a cancer of the lymphatic
________________________________________ system, and bronchogenic carcinoma is a lung
4. cancer. Actinic keratotic lesions are precancerous
A term used to describe discolored areas of aging skin lesions.
skin due to damage to capillaries is: Integrated Process: Nursing Process; Analysis
Your Answer: Actinic keratosis. Cognitive Level: Knowledge
Correct Answer: Senile purpura. NCLEX-RN Test Plan: Physiological Integrity;
Reduction of Risk Potential
Rationale: During the normal aging process, ________________________________________
the vascularity of the dermis decreases and 6.
contributes to a paler complexion in the white- An elderly client is being assessed because of a
skinned older person. The capillaries become rough, scaly, erythematous papule on the patient's
thinner and more easily damaged, leading to body. Important teaching for this patient would
bruised and discolored areas known as senile include that this is most likely a (an):
purpura. Your Answer: Erythematous actinic keratotic
Integrated Process: Nursing Process; Assessment lesions.
Cognitive level: Application
Rationale: Erythematous actinic keratotic NCLEX-RN Test Plan: Physiological Integrity: Basic
lesions are the most common type of precancerous Care and Comfort
lesion. They appear as painful rough, scaly, ________________________________________
erythematous papules or plaque. 8.
The acrochordon, seborrheic keratoses, and senile In order to prevent shearing injury to a 50-year-old
lentigines do not become malignant. terminally ill patient who is high-risk for skin
Integrated Process: Teaching/Learning damage, which of the following measures would be
Cognitive Level: Analysis effective? (Select all that apply.)
NCLEX-RN Test Plan: Physiological Integrity; Your Answer: Keeping the patient's skin clean
Reduction of Potential Risk and dry
________________________________________ Correct Answers: Encouraging the client to use a
7. trapeze bar
Which of the following factors affect skin tolerance
and enhance the occurrence of pressure ulcers? Keeping the patient's skin clean and dry
(Select all that apply.)
Your Answer: Decreased serum albumin Rationale: The high-risk patient for skin
Correct Answers: Shearing damage will need frequent position changes and
clean, dry skin. A trapeze bar, turning sheet, and
Increased body weight transfer board are specific interventions that can be
used to prevent shearing injury to the skin.
Dramatic loss of collagen Integrated Process: Nursing Process; Planning
Cognitive Level: Application
Rationale: Shearing, or the sliding of parallel NCLEX-RN Test Plan: Physiological Integrity; Basic
surfaces against each other, occurs most commonly Care and Comfort
when the client slides down in the bed. Increased ________________________________________
body weight and dramatic loss of collagen also 9.
could increase the risk of pressure ulcers. An Which of the following statements is an accurate
increased serum albumin would not create a skin description for the inflammatory phase of wound
problem, but a decreased serum albumin would development? The inflammatory phase:
indicate a low level of protein stores and increased Your Answer: Begins soon after injury and
risk for impaired skin integrity. continues for up to three weeks.
Integrated process: Nursing Process; Analysis Correct Answer: Is characterized by redness,
heat, pain, and edema or swelling. nursing practice?
Your Answer: They are the legal standard for
Rationale: The inflammatory phase of healing gerontological nursing practice.
is characterized with the classic symptoms of
inflammation, such as redness, heat, pain, and Rationale: The Scope and Standards deal with
edema or swelling. The other answers relate to the the professional obligations of the nurse when
different phases of wound development. caring for the elderly; they are not intended as
Integrated Process: Nursing Process; Assessment assessment measures. Boundaries of practice are
Cognitive Level: Knowledge addressed in the Nurse Practice Act of each state.
NCLEX-RN Test Plan: Physiological Integrity; Basic Nursing Process: Implementation
Care and Comfort Cognitive Level: Analysis
________________________________________ NCLEX-RN Test Plan: Safe, Effective Care
10. Environment
A patient develops severe inflammation of the lower ________________________________________
extremity. Which of the following foods should be 2.
included in the diet for this patient? When an elderly client refuses to take the usual
Your Answer: Carrots and other yellow morning medications or get out of bed, the nurse
vegetables must deal with ethical issues related to:
Correct Answer: Citrus fruits, protein, and fluids Your Answer: Veracity.
Correct Answer: Autonomy.
Rationale: The diet of the client with
inflammation should be high in vitamin C, to aid in Rationale: Autonomy relates to the client’s
the synthesis of collagen; protein, to aid in the right to self-determination and right to make
formation of blood cells and tissue; and fluids, to decisions. Beneficence has to do with doing good,
remove metabolic waste and rehydrate the client. veracity deals with being truthful, and justice has to
Integrated Process: Nursing Process; Planning do with being fair.
Cognitive Level: Synthesis Nursing Process: Assessment
NCLEX-RN Test Plan: Physiological Integrity; Basic Cognitive Level: Application
Care and Comfort NCLEX-RN Test Plan: Safe, Effective Care
Environment
1. ________________________________________
How do the Scope and Standards of Gerontological 3.
Nursing Practice (ANA, 2004) guide professional What is the advanced practice registered nurse
(APRN) who is board certified (BC) in gerontology NCLEX-RN Test Plan: Psychosocial Integrity
most likely able to do? ________________________________________
Your Answer: Serve as an administrator of a 5.
nursing home. With what part of the functional health pattern
Correct Answer: Be employed as a nurse assessment is the nurse most concerned when an
practitioner. elderly female is admitted to the hospital with deep
jaundice?
Rationale: This role requires a graduate Your Answer: Nutritional-metabolic
degree, and those with Master’s degrees may seek
board certification. Case managers are not required Rationale: Jaundice is caused by obstruction
to be nurses, and nursing home administrators are of one of the biliary ducts, and most frequently
required to have specialized education and results in appetite changes. Health perception-
certification in that discipline. Researchers are health maintenance and coping-stress management
qualified to develop and test theories. do not address functional changes. Elimination is
Nursing Process: Implementation not a priority concern when jaundice is present.
Cognitive Level: Analysis Nursing Process: Assessment
NCLEX-RN Test Plan: Safe, Effective Care Cognitive Level: Application
Environment NCLEX-RN Test Plan: Health Promotion and
________________________________________ Maintenance
4. ________________________________________
What information will the nurse use to respond to 6.
the elderly client who states, “I will never go to a An appropriate goal for a elderly male who had a
nursing home; I am going to die at home”? CVA with right-sided hemiplegia and a nursing
Your Answer: “Most people will need a brief diagnosis of impaired self care management would
stay in a nursing home at some time in their lives.” be:
Your Answer: Requests that wife shave him
Rationale: Current data indicate that over daily in the morning.
40% of people will need at least a brief stay in a Correct Answer: Communicates need to use the
nursing home to recover from surgery, illness, or urinal.
injury at some point in their lives. Other responses
do not provide correct or supportive information. Rationale: Requesting to use the urinal is
Nursing Process: Implementation indicative of self-care. Other options do not deal
Cognitive Process: Application with the client’s managing his physical care by
himself. involve the client. Assessment must occur prior to
Nursing Process: Planning setting goals. Implementation is based on the
Cognitive Level: Implementation outcomes; evaluation follows implementation and
NCLEX-RN Test Plan: Physiological Integrity determines if outcomes have been achieved.
________________________________________ Nursing Process: Planning
7. Cognitive Level: Application
The nurse expects to use the nursing process when NCLEX-RN Test Plan: Physiological Integrity
first meeting an elderly client in the clinic. What is ________________________________________
the priority nursing action before initiation of the 9.
nursing process? Allowing an elderly female hospitalized for surgical
Your Answer: Ascertain the chief complaint. repair of a fractured hip to take her medications
Correct Answer: Establish rapport with the client. with her morning cup of coffee is an example of:
Your Answer: Providing culturally acceptable
Rationale: Effective use of the nursing care.
process is dependent on the nurse first making the Correct Answer: An appropriate nursing
client feel comfortable and creating an environment intervention.
of trust. Other actions would be appropriate after
rapport is established. Rationale: Clients can select the fluid of
Nursing Process: Planning choice for taking medications unless there is a
Cognitive Level: Analysis specific known contraindication for a particular
NCLEX-RN Test Plan: Safe, Effective Care medication. This intervention is not related to
Environment cultural practices or to evidence-based practice.
________________________________________ Case management deals with planning for the
8. hospital stay and post-hospital care needs.
After providing basic care for an elderly client, the Nursing Process: Implementation
nurse concludes that hydration is a priority need. Cognitive Level: Analysis
This decision represents which step of the nursing NCLEX-RN Test Plan: Physiological Integrity
process? ________________________________________
Your Answer: Assessment 10.
Correct Answer: Planning Current demographic trends project a steadily
increasing elderly population. What research focus
Rationale: Setting goals or outcomes is the recognizes this trend?
planning phase of the nursing process, and should Your Answer: Studies that demonstrate the
best timing to introduce the topic of durable powers
of attorney for healthcare
Correct Answer: Studies that examine the role
strain of caregivers for the elderly

Rationale: Increasing numbers of elderly


mean there will be increasing numbers of caregivers
who will experience stress. Studies need to be done
to find ways to reduce the stress those individuals
experience, whether they are family members or
healthcare professionals. Other options deal with
specific situations that are not related to population
trends.
Nursing Process: Evaluation
Cognitive Level: Analysis
NCLEX-RN Test Plan: Safe, Effective Care
Environment

You might also like